Quantum New

You might also like

Download as pdf
Download as pdf
You are on page 1of 235
CAREER ENDEAVOUR PUBLICATIONS has taken due care in collecting the data and providing the solutions, afore publishing this book. Inspte ofthis, ifany inaccuracy or printing erors are there, CAREER ENDEAVOUR PUBLICATIONS ‘owes noesponsiblly. CAREER ENDEAVOUR PUBLICATIONS willbe gratefulfyou could point out any such ero. Your suggestions willbe highly appreciated ©All right reserved by CAREER ENDEAVOUR PUBLICATIONS. No part of this book may be reproduced cr utilized in any form without the written permission from the publisher. Preface to the First Edition Quantum Mechanics form an important part of the syllabus for the graduate and post- graduate physics courses of various Indian Universities. While teaching this subject, particularly in physics honours classes, we very often refer to various standard books for the various topics of Quantum Mechanics. But none of these books is fully sufficient for the preparation of different competitive examinations at national and state levels such as CSIR-UGC-NET/JRF, GATE, TIFR, JEST, BARC etc. In all these competitive examinations, the section of Quantum Mechanics catties a high percentage of weightage of the total marks as follows: ‘+ Inevery year CSIR-UGC-NET paper contains questions of at least 35 to 40 marks from this section out 200 marks. + Inevery-year GATE paper contains questions of atleast 12 to 15 marks from this section out 100 marks. ‘The purpose of the present book is to provide a brief discussion of every importanit topic of the Quantum Mechanics syllabus. Each chapter contains a significant number of solved problems some of which contain subtle points, not explicit in the text and’hence will help the students to attain a deeper understanding of the ideas presented in the book. Moreover, the problems that are asked in CSIR-UGC-NET examinations till date, has been solved at the end of each chapter. Besides this, problems that are asked in GATE, TIFR, JEST till date, has been given at the end of each chapter for the practice of the students. This will surely help the student to enhance their self-confidence. Itis hoped that the book will be able to meet up all the requirements of the students preparing for the various competitive examinations: However, any constructive criticism and suggestions for the improvement of the book will always ‘be highly appreciated. Much care has been taken to minimize the typographical errors, yet if any noticed, then your suggestions are invited with a view to improve the book in future editions: The author (Surajit Paul) would like to thank my parents, family members, students, friends and specially Anwesha Chakraborty who have given constant support in the development of the book. Thanks are due to the unit of Career Endeavour Publications for making sincere efforts towards timely publication of the book. Thanks to Mr. Surajit Kuila for his constructive support in doing corrections and proofreading, Last but not the list, a very special thanks to Mohammad Firoz for typing, formatting and designing of the book. CONTENTS CHAPTER 14 12 13 14 CHAPTER 24 22 23 24 25 26 CHAPTER 3a 32 33 34 35 36 CHAPTER. 44 42 CHAPTER Sa 52 83 54 55 : Origin of Quantum Mechanics Introduction Particle aspect of radiation Wave nature of matter Heisenberg’s uncertainty principle CSIR-NET/JRF Previous Years Questions with Solutions Practice Set : Concept of Wave Function Introduction + Physical significance of wave function Hilbert space Operator formalism Schrodinger equation Various representation of wave function CSIR-NET/JRF Previous Years Questions with Solutions Practice Set One-dimensional Potentials Introduction to bound states One-dimensional infinitely deep potential well One-dimensional symmetric finite potential well One-dimensional step potential One-dimensional rectangular potential barriet One-dimensional linear harmonic oscillator CSIR-NET/JRF Previous Years Questions with Solutions Practice Set : Angular momentum & Spin Orbital angular momentum Spin angular momentum CSIR-NET/JRF Previous Years Questions with Solutions Practice Set : Three-dimensional potentials ‘Three-dimensional rectangular box Three-dimensional cubic box Three-dimensional harmonic oscillator Spherically symmetric potentials Hydrogen atom CSIR-NET,/JRE Previous Years Questions with Solutions Practice Set rr 14 a 22 24 25 26 27 37. a 44 55 55 64 70 75 80 92 102 110 115 127 133 138 ui 144 147 149 158 164 CHAPTER 6; Identical Particles 61 “62 63 64 Introduction Symmetrization postulate Symmetric and anti-symmetric wave function (space part) ‘Symmetric and anti-symmetric wave function (spin part) CSIR-NET/JRF Previous Years Questions with Solutions Practice Set CHAPTER 7: Perturbation Theory 74 72 ‘Time independent perturbation theory Time dependent perturbation theory CSIR-NET/JRF Previous Years Questions with Solutions Practice Set CHAPTER 8: Scattering Theory a 82 83 84 8s Introduction Scattering cross-section Scattering amplitude First Born approximation Partial wave analysis CSIR-NET/JRE Previous Years Questions with Solutions Practice Set CHAPTER 9: Variational and WKB method on 92 Variational method WEB approximation (Wentzel-Kramers-Brilouin Method) CSIR-NET/JRF Previous Years Questions with Solutions 169 170 171 172 176 178 179 192 198 206 209 209 210 214 213, 216 218 220 224 227 Origin of Quantum Mechanics 1.1 Introduction : tthe end of the 19th century, physics consisted essentially of Newton’s Classical mechanics and Maxwell's theory of electromagnetism. Classical mechanics was used to predict the dynamics of material bodies and ‘Maxwell's electromagnetism provided the proper frame work to study radiation. In fact, matter and radiation were described in terms of particles and waves and the interactions between matter and radiation were well explained by Lorentz fyyee x o ss of classical physics, classical elec- F been acheived and every physical ity of Newtonian mechan plaination for several newly dis- nthe em. wave of frequanecy of energy. for light and| posted that light explained the photo logtrigt both the theoretical sh a microscopic level. in ition exhibits paticlelike behaviour but conversely mater 1cept was experimentally confirmed by Davisson and po in yegareerendeavour. Cem Generally there are two independent formulations ofquantum mechanics. The frst formalation called matrix iechanicS) was devoloped by HESEABEAEIAII25 and Second formulation called wave mechanics, was devoloped by Sehrodingerin 1926, which is the generalizationofDe=brOglie postulate. These two different formulations of quantum mechanics were shown to be equivalent. Moreover, Dita® bas suggested a more ‘general formulation of quantum mectanics which dais With @bSHaCTObjeGts like kets (State) VECtOrS)) bras jand operators. The representation of Dirac formalism in a continuous basis i. in the position and mo- ‘mentum representation gives back the Schrodinger’s wave mechanics and the representation of Dirac formalism in a discrete basis gives the Heisenberg’s matrix formulation. ‘According to classical physics(@lpaifibleis characterized by an @HERVID and a AiOEHERTIDS whereas AWANE is characterized by an éimiplitide and Wave vector © that specifies the direction of the propaga- tion of wave. For instance, the particle and wave properties are mutually exclusive and wave can ex- change any amount of energy with particles. Now, we will see that how the phenomenas like photoelec- Origin of Quantum Mechanics " Photoeletric Effect: During his experiments on e.m. waves in 1887, Hertz noticed that sparks occurred more rapidly in the air ‘gap of the transmitter when an ultraviolet light is directed at one of the metal balls used in that experiment, Phillip Lenard, a student of Hertz studied the phenomena. He soon discovered that when the frequency of light is sufficient'y high, electrons are emitted from the metal surface. ‘The phenomena in which electrons are emitted from the metal surface (e.g. Na, K, Csetc,) when iradiated with light (photon) is called “Photo-electric effect”. The metals which exhibits this phenomena are called “Photo-sensitive materials” and the emitted electrons are called “Photo-electrons”. The existence ofphoto electric effect is not surprising. Light waves carry energy and some of the energy absorbed by the metal gets concentrated on a particular electron and reappear as their kinetic energy. Lenard’s Experiment : Lenard’s experimental arrangement is as shown in figure above. Two plane metal plates C and D are ‘mounted in an evacuated tube. Plate G is coated with photosensitive metal and maintained at a negative whereas A another metal plate and kept at zero potential, The beam of negative ions emitted from C can pass through a hole in A. UV-radiation is incident on C through the quartz window W fitted to the side tube Gand negative ions are emitted, These are attracted to Aand apart oft pasing through the hoe in Aiscollated at D. These lectrometer E. where, m is the mass Further, apo paga ale is foupdito decrease with theincrease of positive potential UINAANEMs Sa ,) the current becomes zero. Then, this V, is called stopping potential, Electrons of al velocities up to the maximum y, are stopped. fromteaching 4. Then Lye Ex.inix 25 ax = Vs ae Some experimental observations: " ° Im A ° Vo Figure : Variation of photo current Figure : Variation of photo current () with the intensity of radiation (1) (i) with Potential difference (V) nice heair acyof sated zalled photo metal Dare ative Cean eside olein sseof zero. "ped Origin of Quantum Mechanics ee _ 7 an Vs; Ven Vs, ae Ye Ym My V Figure : Plot of photocurrent (i) against Potential : difference V showing the variation of stopping Figure: Plot of E,,, against v. ‘potential (V,) with frequency () of radiation. (There is a certain minimum frequency v, of incident radiation below which there is no emission of photo-electrons regardless of the intensity of the incident radiation. This frequency is known as threshold frequency which depends on the material and the nature of the emitting surface. (i) The maximum velocity or kinetic energy of the emitted photoelectrons depends on the frequency of the incident light, not on it's intensity ie. biue light results in faster electrons themed light. The kinetic energy of the ejected electron increases linearly with incident frequency. (ii) The emission of photo-electrons i instantaneous i.e. no time interval between arrival of light at the é i le the incident light is, (iv) Ata fixed freqie i j i = the intensity ofthe light but ( According to of threshold f L a Gi According to wale g uted over the entire wave front, frequency oflight, no concept So, electrons w cept of instantaneous emission. (iii) The electric fi E { forcesta liberate clectrons from the metal surfag er fields and should give higher velocity to the ' Einstein’s hypot anta of energy & = hv. ii) Energy in light if , ¢ wavetro ‘oficentrated in small packets known as ‘photons’. suetal sf ‘he whole energy to one electron without any time’ (iil) A certain mininghA RAE OBNG OF OKADE WV OG NsGQTY Ps to release an electron from the metal surface Photo-clectric equation: When a radiation of frequency v isincident on the metal surface in vaccum, it canbe either be reflected from the metal surface or it can be completely absorbed by an electron. Inthe second case, the electron can be knocked out of the surface. The emission of electron from the metal surface is possible only if hy > Wy. The remaining energy (iv-Wy) canbe utilized to impart Kinetic energy ofthe emitted electrons. Figure: Mechanism of Photoelectric effect. Soln. Soln, Soln. iS Origin of Quantum Mechanics So, the maximum Kinetic energy of the emitted electrons is 1 Ex guax = MV aay, = AV — We = hv — ho 2 where ‘m’ is the rest mass ofthe electron, vq, is the maximum velocity of the emitted electrons and V, is the threshold frequency. This is the well-known Einstein's photo-electric equation, Explaination of Photoelectric effect: ( Ifv 2).The radiation at wavelength 4 is called unmodified radiation whereas radiation 2 at wavelerigth is called modified radiation. This phenomenon is known as Compton Effectand the wavelength difference A’ =A is called Compton shift. This shift is independent of wavelength 4 of the incident beam and the nature of the scatterer but depends on the angle of scattering. Experiment: ‘A beam of monochromatic X-rays is allowed to fall ona block of graphite and the intensities of thescatiered X-rays are measured as a function of wavelength of X-rays, at different scattering angles, gyObserver Foreach angle of sc fo aiprent wavelengths, Figure : Variation ofintensitis of scattered light as a function of wavelength for various scattering angles (#). ae Origin of Quantum Mechanics Failure of classical theory: Inclassical wave theory, X-rays aree.m. waves. If v is the frequency of the incident X-rays on the material, then they cause electrons in the material to oscillate at the same frequency v. Hence, the scattered X- radiation should consist of single frequency (or single wavelength 2 ). gain, electrons should radiate X-rays uniformly in all directions and hence, wavelength of the scattered radiation should not vary with scattering angle. Assumptions: Compton applied Binstein’s quantum theory of light with that assumption that the incident photons posses ‘momentum in addition to energy. 1, The beam of incident monochromatic X-rays of frequency v consists of streams of photons each of h y and momentum P=""", traveling along the direction ofthe beam with velocity ‘c 2. The scattering is due to the elastic collision between photon and the fie electron at rest. Hence, the conservation laws of energy and momentum bold good in the scattering process. energy ¢ Explanation: We considera beam of monochromatic X-rays of iequency v and wavelength 2 beincident on target electron. Let, the incident: : angle ¢ and the electroff Ifthe frequency ofthe: Therefore, the kinetic ef wunvcaresrendeavourct com qi-@/o ~Q) Applying law of conservation of energy: hv = hv’ + E, Applying law of conservationof tinear momentum along and perpendicular tothe direction of incidence: ~Q) From the equation (3) we can obtain the compton shift to be, h 2azat=—L(1-cosp)=a,(I-e A-A=Ah at cos) = A, (I-cos¢) @ 07496 4° i¢Comnton wavelenath where. 2 ea wnics Origin of Quantum Mechanics MES @ Conclusions: terial, ( Compton shiftis independent of the wavelength of incident radiation and of the nature of the scattering dX material. “rays (i) Compton shift depends on the scattering angle @. tering (Gi) Largeras the rest mass of the scatterer is smaller. Relation between vandv': “ =1+—" (1 cos¢)=1+a(1~cos¢) posses v mee’ ach of Relation between @ and g: cot =(I+a) tan’ Kinetic energy of the recoil electron: 2e, the target length. aghan 1ap the intensities of unmodified X-rays Soln. sively, Son. ‘Example 12. A beam of 7 -radiation having photon of energy 510 keV is incident on a foil of aluminium. Calculate the wavelenth of radiation at 90° and also the energy and direction of the emission of the corresponding electron. he Energy of a photon of frequency vis €= hv a Here, e= 510 keV =510x10° x1.6x10-9 7 he __ 66x10" x3x108 € 510x10°x1.6x10- The wavelength ofthe scattered radiation is, a =2.426x10°? m Origin of Quantum Mechanice Ori ioe rie a+ (-cosg) 2.426 x107!* +6.6x10 Uses) moc 9.1x1079! x3x3x10 = 2.426% 107)? + 2.417107? = 4.843% 1071? m The energy of the recoil electron is, B=hv-hv'en nv) = AAD 6.6010" x3%108 x (4.843107! —7.426x10""*) 2.426%10"}? x 4.84310 ‘The direction of the recoil electrons is given by, 72x10 47 cot 45° 1 1 140.9985 1.9985 6.61074 Soln. —— 114. Aphoton of energy ¢ is scaticl ‘a free electron of mass m, and observed at scattering Ta éareerendeavourcom angles greater than %. Show tat the energy &" ofthe scattered photons ks than 2mgc?, however large the value of € maybe. Soln. ‘The energy of the scattered photon is, e e 1+2asin?® 142 sin? S 2 met 2 28 x For scattering angle ¢> pans #oSsintg>tat Therefore, the energy ofthe scattered photon is, > das sing is an increasing function of ¢. For any value of. = where 421 according as @2 7. anice Origin of Quantum Mechanics ttering ‘eethe lue of = € € mie” —s—- Lt 26 Eee meg mye? 4 2nige’ E : 7 mpc? Since, positive quantity and f > 1, “0+ isabways greater than {and ¢isabvayslessthan 2mgc? , however large the value of ¢ maybe. 1.3. Wave Nature of Matter: According to classical mechanics, the particle can behave as particle and wave can behave like a wave. Particle is characterized by energy and momentum and wave is characterized byamplinude and wave vector. In 1923, De-broglie postulated that a moving particle can behave as a wave. The de-broglie wavelength associated with a particle of est mass ‘m,’ moving witha velocity ‘v’ is given by where ¥ is Lorentz factor 927. Ifthe De-broglie wavekngth can be experimentally detected. Otherwise, ifthe Dey a ot be detected experimentally Schrodinger modi a group of waves Phase Velocity: where, o isthea The phase ¢ ofthe wave a ‘any time e eames #(3)= aa emer ter WWMNSGATEPLRIGEAVOUL COM. gcocesererese also the velocity of a monochromatic wave with which crest or trough of the wave travels ina medium Ina dispersive medium, v, is different for different wavelengths. The phase velocity ofthe propagating ménochromatic ere aiveis givenby, Yp ==. waves gi ean From the relativistic mass energy relation, y, == = _ ¢ “Pp ymy ov Since, the phase velocity of'a de-broglie wave is always greater than the velocity of light in freespace. But according to the special theory ofrelaticvity, no energy can be transmitted with velocity greater than ¢. This suggests that the phase velocity has no physical significance and we can not respresent the wave nature of moving particle by a single plane wave. Origin of Quantum Mechanics Group velocity: os] A wave group or wave packet is described as the superposition of individual waves of different wavelengths These waves interfere with each other and results in variation of amplitude, These defines the wave group shape. If the medium is non-dispersive, then phase velocity of the individual waves will not change with wavelength. Asa result, wave packet travels with common phase velocity. But, ina dispersive medium, phase velocity of individual waves will be different. As a result, wave packet moves with a different velocity thax phase velocity which is known as group velocity ‘,’. Let, the wave group is formed by the superposition oftwo monochromatic waves having same amplitude but v5 < vp av, (@Anamolous dispersive medium: “7 =-ve=> v, >, Example 15. What Voltage must be applied to an electron microscope to produce electrons ofwavelength 0.40A? der a potential of V votsis, 1 Thede-Broglie wavelength 2 ofaparticle is 4 oc —L. and fora photon 4% {F Thisshows that de-Broglie VE wavelength of an electron is much smaller as compared to that ofa photon for the same energy E. Example 18. Energy of a particle at absolute temperature T is of the order of kT. Calculate the wavelength of thermal neutrons at 27°C. [Given, the mass of the neutron =1.67%10-*4 gm. Planck’s constant, k=6.67x10" erg sec and Boltzman constang, k =8.6x10~> eVk] ‘De-Broglie wavelength of the neutron is h h Az = t= =1:781° V2mE V2mkT Soin. Origin of Quantum Mechanics Example 19. Abeam of monenergetic neutrons corresponding to 27°C is allowed to fallona crystal, A first order reflection is observed at a glancing angle 30°. Calculate the interplanar spacing of the crystal, Given, Planck constant, h= 662x104 Js Mass of neutron, m=1.67x1077 kg Boltzmannconstant, — k=1.38%10 J/K FromBragg law of diffraction we have, where, the glancing angle, = 30°, order number of diffraction, n= 1 2d sin30°= A, or, 2d x. ,or,d=d 2 De-Broglie wavelength ofthe neutron is h h de ZO\I MeV using both relativistic and relativistic expression for 4? 14. Hoisenberg’s WMA TeRERNGeAVvoUr.cOM According to classical mechanics, a moving particle has a definite momentum and position in space. Both the position and momentum of such a particle can be definitely determined. Recently, the classical viewhas been proved to bean approximation and is inadquate for describing the dynamical behaviour of the macroscoy objects. It cannot describe satisfactorily the behaviour of sub-atomic particles. In quantum mechanics, a particle is described by a wave packet of linear extension (Ax) which surrounds the position ofthe classical particle and moves with grou velocity. According to Max Bomn’s probability interpre- tation, the particle may be found everywhere within the wave packet. This means that the position of the particle is uncertain within the limits (Ax) ofthe wave packet. So, we may say that the particle may be found anywhere within (Ax). Again, a wave packet is formed by the superposition of large number of waves having different wavelengths, the wavelength the wave packet cannot be said definitely. What we can say that the wavelength ofthe wave packet lies within certaif range 4 to 4+ .Asthe momentum of a particleis related to wavelength by i 1 { ; os first | Origin of Quantum Mechanics = ie) h Gaal AA the particle momentumis also withinp to p+ Ap . So, we cannot definitely say what is the momentum of the particle. Hence, because of the wave nature of pastice there is uncertainty (Ax) in position of the particle as well as uncertainty in corresponding momentum (Ap) . Ifthe number of superposed wave as well as their range of wavelength (A2) be increased, then the linear speed (Ax) ofthe wave packet decreases. But simultaneously the uncertainty (Ap) in momentum increases as Ap x AZ. When AA ==, Ap =o and Ar=0, ie, the wave pocket reduces to a point. Inthis position, there is no uncertainty in the position of the particle. Now foradefinite 2=0, 4 =0,then Ap=0(-" Ap o AZ) ,thenthere isno uncertainty inthe momentum of the particle but in this case Ax =00, ie, the position uncertainty is infinite since the associated wave with the particle may now extend up to «0. Statement of the wiée The uncertainty p Je sumultancously the position action. 104 Js. Here Ax is the error or (uncertainty) inthe determina fely then Ax becomes smaller and ertaintes arenot inthe appratus ee © Hax=0, the, Wnnaiscat Seferideavour.co This means that, ifthe uncertainty inthe position measurement ofa moving particle be zero, then atthe same moment the uncertainty in the momentum measurement willbe infinity. (b) If the momentum ofa moving particle at any instant be determined accurately (1.¢., Ap, = 0), then the uncertainty (error) in the determination of position of the particle willbe infinite. (©) Fora particle of mass ‘m’ moving with velocity v, along the x-direction, its momentumis p, =mv,, For aheavy particle mp andin that case, AxAv, For sucha patticle both the position and momentum can be determined accurately. Thisis true for macroscopic bodies whose motions are discussed in classical mechanics. Thus the uncertainty relation has no meaning in classical mechanics, But for microscopic objects such as electron, proton, atom, etc, quantum mechanics is annlicahle and acench the nncertainty relation ic reality in decorihina the damaminal hahavinwrafench ahiacte q@ Origin of Quantum Mechanics ‘Time-Energy Uncertainty Relation: ‘The time-energy uncertainty principle is stated as follows: Inany simultaneous determination of time and energy of a moving particle, the product of the uncertain- ties is equal to the Planck’s constant, AEAt>h Here, Ax is the uncertainty in the measurement of energy of Ar, the corresponding uncertainty in the measurement of time. If AE be the maximum uncertainty in the determination energy of a system in a particularstate,then At isthe ‘minimum, ie., the system remains for a minimum time in the state. On the othex hand, ifa system remains ina particular state for a maxinium interval ofime, then the uncertainty in the measurement of energy s the mini- mum, Angular momentum- angular position Uncertainty Relation: ALAG>h Where, AL is the uncertainty in the angular momentum ofthe particle and Ag isthe uncertainty in the angular position ofthe particle. agi Causality in Quantufh M166 instant is sufficient for the prediction ofits fut assical principle that, the future behaviour ofa particle at a given instant. The law of} causality, however, ho Sn be made without disturbing a rmicrosystem, it does According to Max /0 alternate angles. In wave- described by a wave function e behaviour of the system ata | point P(7) at any tis shows that even for cannot be carried out | system unambiguousy Ee did of the system unambiguowsly at t= 0. Thus the prineiple of eats f evens ren be adopt a acearange eh tere } Wwww.Careerendeavour.com i Concept of Bohr Orbi view of uncertainty principle: Wehave the energy-time uncertainty relation, AE.At> A According tothe concept of Bohr, each electron revolves around the nucleus in quantised stationary orbits with definite energy E. So, the uncertaintyis AE =0. is aE 0 | Hence, all energy states of the Bohr atom must have infinite lifetime (r =o) . This implied that radiation | So, would not occur due to the transition of electrons, But actually Ar ~ 10~* sec leading to the occurrence of | emission of radiation due to electronic transition. Thus the concept of Bohr orbit violets the Heisenberg’s uncertainty principle. i cs ith’ Origin of Quantum Mechanics el G@ h Again, if At is finite, then AE = is also finite, This means thatthe energy levels in the excited state must have a finite spread. As a result the radiation emitted due to the transition ofan electron fromthe excited state to the ground state is not monochromatic. This leads to the conclusion that the spectral lines can never be ‘infinitely sharp but must have a natural broadening. This is contradiction with Bokr’s theory of spectral series, Electron difraction through a single slit: Let a monoenergetic parallel beam of electrons moving along the x-direction be incident ona single split of width Ay. As the electron in motion is associated with de-Broglie wave, the electron beam after passing through the slit undergoes dftaction producing a raunhofer diffraction pattem onthe photaplateperpendicular to the x-axis, é shebeerainty inthe position ofanelectron intheslitis Ay asit Td Enh gélegtron in the slit, CAREER ENTER Before incidence, AWWRAAM BARONE MAGA NORM UGG tion be p. Aer diftation ifthe electron moves along OC, then the component of momentum p along the y-direction is p sin @ . Since the electron may be anywhere within -@ to +6, the y-component of momentum must be anywhere between =psind to psind. ‘Thus the uncertainty in the momentum measurement of the electron is, dpy= psind~(-psind)* 2psind =24sin 4th Therefore, AY-Apy = ax To decrease the uncertainty inthe determination position of the electron, Ay must be decreased. But the above equation shows that a decrease in Ay leads to the increase in g and the diffraction pattern will be wider. This will cause a larger uncertainty in Ap . Thus the electron diffaction experiment established the truth of uncertainty principle. sind =2h2h Soln, Soln. Origin of Quantum Mechanics ‘Consequences of Heisenberg’s Uncertainty principle: (1) Non-existance of electrons within the nucleus: Suppose electron exist within the nucleus, then uncertainty ih the position of nucleus willbe ofthe order of twice ofradius ofthe nucleusi.e, Ax =2x10"m. ‘Then the minimum energy ofthe eleetronis given by Eh, = piel tie! = Ey = (Ap) c? + mic* a = Bay = (4) ct +mic! ~10MeY 2a Therefore, ifelectron exist within the nucleus, then minimum energy of the electron will be approximately 10 Mev. But, experimentally energy comes out to be of the order of 4 MeV. Thus, we can say that electron does not exist within the nucleus. (2) Ground state energy of a linear harmonic oscillator: Leta particle ofmass mis executing SHM along x-axis and let at any t position, Therefore, i Total energy of LHO ‘t itspositionis‘x’ fomthe mean For minimum energy, ‘ni il So, the minimum ener [7 Merrie parvo meeiinies poe snand momentum] a_ WWW Careernt deavour.com ma my ie mt On my ‘Example 22. Suppose that the momentum ofa certain particle can be measured to an aecuracy of one part in 1000. Determine the minimum uncertainty inthe position ofthe particle ifthe particle is ()a 5x10 4g mass moving witha speed of 2 m/s; (i) an elecron moving with a speed of 1.8108 m/s Given: 222107 = ap =107%p=10°3 mv P Therefore, the minimum uncertainty in position ofthe particle will be Origin of Quantum Mechanice h y-20 40 2) pug a = 5.28107 Onin ™ 4nmgyxl0> hyl-v/e? i) (Ax) pig = @ (A) nia 4mmgvx10> Example 23. The average time interval that elapses between the excitation ofan atom and the time it radiates 710 energy is 19° sec. Calculate the uncertainty in the energy of emitted photons and the limit of accuracy with baad which the frequency of the emitted radiation may be determined. Soln. Uncertainty in energy of the emitted photons willbe haataxioev ar ean Soin. th, . ou Example 25. In fatom is produced by vapourising silver salt in a furna inty principle to find the order of magnitude of the sme Gr FERENCE 2 Melo atom = 1,66%10"*7 kg] Soln. Ifthe silver atom of mass ‘m' moves with velocity ‘v' along the x-axis from the furnace at temperature T, then its kinetic energy is, mv? ser tin m = Time required by the silver atom to travel a distance of 100 emis, 200 _ 100 V3KT Im AE At =A => (AE) iin (Mt) ng 2A => (AE) rin * i | " Now time-energy uncertainty relation is, in Soln. Soln, Origin of Quantum Mechanics wow (At? = AE p= gg = 2 ‘Maximum uncertainty inthe position of the particle 7, ag = 7107 m = 0.7m 1. [Be arr (#) The smallest spot on the detector corresponds to the maximum uncertainty in position ofthe silver tom. Hence, size of the smallest spot=0.7 um, Example 26. The speedometer ofa 180 kg vehicle reads 72 knw/h ata particular instant, The readings are ‘known to have an accuracy of +2 knv'h. What would be the minimum uncertainty in the position ofthe vehicle at that instant avcotding to the Heisenberg uncertainty perinciple? ‘The uncertainty in the momentumis Aps = A(mv,) = may, Thus the minimum wi Remember the size of snatem is off Gps faster is 10m Asize of 1 mistoo Areit. ; Fe nuclei make transition from is about 3104s. Ifthe spread in vacuum, From energy-time 2 | Origin of Quantum Mechanice @ CSIR PREVIOUS YEAR QUESTIONS Qt. Which of the graphs below gives the correct qualitative behavior ofthe energy density £,(2) ofblackbody radiation of wavelength 2 attwo temperatures T, and; (T, 7;). Theenergy emitted per unit time per unit solid angle per unit surface area of a blackbody in the frequency range vto v+ dv is given by B(v)av, Which ‘one of the following graphs has the correct form? [TIFR 2015] BG)| u BO)| © © > 0 v 0 v BO) © www.careerendeavour.com Concept of Wave Function Soln. 2.1 Introduction : There is always a quantity asscociated with any type of waves, which varies periodically with space and time, In water waves, the quantity that varies periodically isthe height of the water surface and in light waves, electric field and magnetic field vary with space and time. For De-broglie waves or matter waves asscociated wat moving itl the quay that vary wih space tne, 3 ‘scalled wave function of the particle, eee finetion y-(348) ) F should be cong pywhere ih pete Butea ous in some cases as follows: {@) Ifthe potential (b) If the potential un ‘of discontinuity at some points, (iii) v(x.) should be; Example 1: Which of for all values of 2? (@) y(2)= Aseox www cateerendeawodr.comy «)=4 y (x)= dsecx and y(x)= Atanx isnot finite at x=, ution of the Schrodinger equation w(x)=de™ isnotfinteat x= 40; y(x)= Ae isfinite everywhere in space Correct option Is (d) Example 2: Aball bounces back offearth. You are asked to solve this quantum mechanically assuming the earth is an infirtely tard sphere. Consider surface of earth as the origin implying V (0) =<» and a linear potential eleswhere (ic. V (x) = -mgx for x > 0). Which of the following wave function is physically admis- sible for this problem (With & > 0): (b) v(x) = de’ ey (x)=naee (8) v(x) = Are b ws: nts, Concept of wave function ! Soln. ie) Since the earth has been taken as infinitely hard sphere, therefore the wave function of the ball willbe zeto for <0, non-zero forx>0 and zero at infinity. Since y (x) should be continous everywhere, the wavefunction of the ball will be zero at x=0. Correct option is (d). 2.2 Physical significance of wave function : Generally, y (x,4) is a complex quantity. It can be mutipliedd by any complex number without affecting its physical significance. In general, y(x,f) has no direct physical significance. But the quantity v' («ty (2.0) =|y(x.0)P is teal, physically significant and is defined as position probability density ie. probability of finding the particle per unit length at time ”. Therefore, for 1-D motion the probability of finding the particle between x to x'+ dr at time ‘? is given by and for 3-D motion the] 3 fige element dr located between F and? +dF at ting Now, v(xA)or vl space should be eq This can be understood as a ‘Method of Noman careerendeavour. com Consider (x,t) is an unnormalized wave function. We can construct a normalized wave function as y(x,t)= Ne (x,t) where N is the normalization constant, Therefore, J (tw thee=|N4|f 6 (x)osthir=1 Jo edeCne Example 3: Normalize the wave function given by v(x)=Ne* (x>0) =Ne* (x<0) =>N Sol. Normalization condition is j v'yde=1=>|NP i enacefea| = a Concept of wave function 1 N= oi [ae tag)" = Orthogonality condition of wave functions : ‘Two wavefunctions y,, (x) and y, (x) are said to be orthogonal to each other, if Ivete Wva(x)de=0(men) i.e. if'a particle isin the state Y, (x), then the particle cannot be in the state y, (xx) simultaneously together. Orthonormality condition of wave functions: Ina vectorspace it pl basis ie. we can express any ‘vector inthis vector sp ined space can be defined in which a set of fiunctions ¢ (x) a coordinate system. The corre- sponding infinite dimets ed inthe interval ax [8 Be] =ih Similarly, [9,A, ]= i and [2,3,]=ih (2) [& Bly = sb ~ Bay Linear Operator: Ifan operator A is said to be a linear if’ ( ALev(2)}=edy (x) and Gi) Aly, (x) +¥2(x)] = Avs (x) + Ay, (x) Ail Quantum mechanical Operators are linear in nature. Properties of the commutator bracket: [as] [ea] of ac a Concept of wave function Soln: Soln: Soln: Solu: 4 [48¢]-[4 8]é+ 8] 42] 1.[7(4).6(4)]=0 10.[ 4",8]=n4™"[4,8] Example4:Find the commutator bracket [[ 4,8], [4,4] [caa}faay)-[aa, a (a ik 4] 2in(xp-+ px) [27] e+ [»[sa]]- Js Ay Eigenvalues and Fig Ian operator § CAREER ENDEAM OUR J), tno v, (0) is cated te cigenfiunction of A corres eae above e pions mas eigenvalue equation not A RRA CPST SAGGANOUI CO ofoperator A. Brample: d=, and y, (2)=a0™ = Ay, (2) -£ (a) =4ae* =4y, (2) Ve r isan eigenfunction of A corresponding to eigenvalue 4. Ifthe commutator bracket of two operators A and B is equal to zero ice. [ A, 3] =0, then the physical observables corresponding to these operators are simultaneously accurately measurable and they havea com- plete sef of simultaneous eigenfianctions. a Example 7: The operator (« + 4) has the eigenvalue or . Determine the corresponding wavefunction. tigenanccsatn: (+L Jy =ay = Wn (a-s)y = [SE flar-x)a stlon vd the sation ysical Concept of wave function Jeinyy sy =yoero{ax-2) @ 2 Example 8: Find the constant B which makes ¢-*” an eigenfunction of the operator (& - Bx ) ‘What is the corresponding eigenvalue? a 5 For e tobe eigenfithétion of the operator (S — Bx? | , then the eigenvalue J (4a*x?~2a~ Bx?) mustbe independent of x ie. (4a*-B)=0= B=4a* 2 Taco —~B = ie rator is (-2a). Schrodinger represdhage 4) Hyde Properties: (1) Eigenvalues off maaan a ‘cdntit ve Hence, the momentum operator , ishermitian in nature. a a. ate Similarly, we can see that — isanti-hermitian ive. | | ~-° Ox. ax. Projection operator: A operator is said to be projection operatorif A' = A and 4? = A Properties: (1) Eigenvalues of projection operator is O and 1. (2) Product of two projection operators is also a projection operator if [6,, p,]=0 B) Sum of two projection operator is a projection operator if { f,, 2} =0 Concept of wave function Soln, Unitary Operator: Anoperator / is said to be unitaryif 4' = 4"! Parity Opertor: Parity opeartor corresponds to space reflection about the origin ie id (x) Ingeneral, Py (*)=y (#); where 7 = xi +y}+ 2 and - When parity operator acts ona wavefunction, the following changes take place in various co-ordinatesystem: () Cartesian co-ordinate system: x —x, y > -y,2 9-2 (i Spherical polar co-ordinate system: r+ 7,0 2-0,>2 + (i) Cyllindrical co-ordiante system: p> p,g>2+$, 2 -z Properties: (1) Eigenvalues of the parity operator is 1, -1 Bu (x)=v(-x)=w(x)v even parity. By (x) =y (-x) =-w (x) > odd parity (Q) Parity operator is hermitian in nature ) Party operator commutes with anion oper ifthe potential under which partclis movingie. V(x) issymmetrc in nat a —— Example 9: Let, N then eigenvalues of N & (a) +veand ~ve integets Example 10: Let the ‘What will be the functipg see” | GAREER, ENDEAVOUR, "www. cateérendeavour.com atte) Wed [24 ota? ‘2m ds| a? nw 4] 24 dig? 24h? ( 2x a =P ta? = 4) 24 x6 a2 |e Se +e 2m axl a? 2ma® | \ a? ‘This isnot an eigenfunction ofkinetic energy. Example 11: An operator Ais definedas Ay (x)=y" (x)y (x)-Isthis a linear operator? Soln. Forthe given operator, Alyi(x)+v2(x)]=[vi(s) +¥2()) [a )+¥2(0)] -vi@)+v5@) [m+] = w(x) (+s (dvr (x) +s Cedys (x) 4y"2 (v2) zion} Concept of wave function = and Ay (x)+ Ayo (x)=¥'i (xvi (x)+¥"2 (xSvu(e) Tus, [yi (2) +2 (x)]=¥'1 (x) + Ay (x) and hence A isnot a linear operator. Example 12. Prove that [py H]= 2p +x [PY] where Hao ay, and V is potential energy stem: operator. 2 2 7 = H\p, = Pray ese Sol. [Xp,, 1] =X[py, H]+[X. H] py al nwt peter, =X [pele ealX, Pel+L%s re] Pe} Px 74/24 ih =X [pal ]+ 55 (2itipr) Pe =o +X[p,¥] Ve) Example 13.|¢,) and |g)) be twe ortonormal state vectors. Let A=|¢))(2|+|¢) (|. Is a projection operator? e : 0; Sola. 4* =[|)(¢,|+ |=4 Hence, Ais hermit ‘Therefore, Aisnot of 7 Example 14. Propet Soln, Gj u Sn (7) (Pa? 2s J+.) (2inX) +(2inX) p,) a Hip,X+ Xp.)=2 (Abe + PX) Gin P= Example 15, Prove thatthe operators i(d / dx) and d? / dx? are Hermitian False oe ity Sut ale 1 Soin. Tua eg )ysteaavivars tf ve Sit ~Flege.) vide Therefore, id /de is Hermition. -dy,) _ dv, dv, jy Ml fave dve gy ly. al Vee Soln, Soln. Soln. a5 a= fo Oey ve] +]v.4 gee Neat de Thus, d? dx” is Hermitian. Example 16: Find the following commutation relations: ace a @ ig S| «i [z . Feo] = oC -2) =0 Ola 7 "oe | laat wa)" lara)” 6. Fy Oy _aF = 2 ryy-Foy oF yer pL, w (2 Fo)|ve8 (Fy)~ Fay ate oe Oe ay as, | 2 -F0)|-% Example 17: Find th o(dlée4 a Therefore, : x & ; a REER‘EA iv. Caréerendeavour.com Example 18, By what factors do the operators (x" p? + p2x*) and 1/2(xp, + p.x) differ? (ple pie) f a ac Of PED 7 wo Sint ( 4 J @ Concept of wave function lon — | concept of wave function Soln, = ~ hon, spake 4) Aldea] oP (pF ae xo 22S x, = (anSt prorSaeT ra Es ay 7 ah ef a(t F 44x 4h = {me Fare, ‘> Bax Se tareasls The two operators differ by a term ~(3/2)h? ‘Example 19. Find the eigenvalues and eigenfuunctions of the operator d/ dx Eigenvalue equation: s d y(x) is the eigenfunction) 2 www.careérente vour.com Note: (1) If the state of the particle is an eigenfunction of the operator 4 , then the expectation value of the physical observable corresponding to 4 willbe equal to the eigenvalue of 4 corresponding to the state of the particle, (2) The state of the particle is given as: 244) Iv)=C)lA)+Calds) +6 |6,)+ are the eigenfunction of the operator 4, then expectation value of the the physical where |6,),/¢2). observable corresponding to A willbe thasinanvaliae nfomarnine 7 where 2 a1 Soin. Soln, Soln, Se = Concept of wave function Example 20: A particle of mass ‘m’ is confined in a 1-D box from x =~2L to x= 2. The wave function ax of the particle in this state is y (x)= vo ow 2) (Find the normalization factor. (@ The expectation value of Pin this state ro ot a(x at 2a: @ J vvae 1 = |v) Jo (Bet =>|vol [ 4[ cos? ad 2 Mo (2x) 4b Yo leah xin 28) AL)" at bl fer 20y] 1h 2a 2 cas? (2) (=) a 4L AL, Example 22, The wave ayooraReheasclekeHOLeReGettqmD. whee N=(/na)". Evaluate (Ax)(4p) Since y is symmetrical about x=0,(x) =0 ( Since, y is a real wave function, (p)=0 (ot) g? fox? (p*)= (tay? fool $=] Soe0[ SE jan ae id a 2f et exp| ldo w fe Ja a Example 23, The Hamiltonian operator of a system is H = -(d?/dx*)+x*. Show that Nyx exp(—x? /2) is an eigenfunction of H and determine the eigenvalue. Also evaluate N by normalization of the function. y= Nxexp(-x? /2),N being a constant (@.) 2 2 2 2 eed 1° oo 5} [oe[-£)-s?e(-5)] Example 24, Con '), where A is a real constant. (i) Find the value of (@ Normalization cf @ (p)=]v" (4) Concept of wave function . maa ate LE) [OOF] Ue s | "ae thik) A? I exp| =cin(-3) ote In the first term, the integrand is odd and the integral is from ~20 to oo. Hence the integral vanishes. Therefore, (p)=Mk, since 4? Joo “art Concept of wave function Example 25. The normalized wave function of a particle is (x) = exp(iax—ibt), where A, a and b are constants. Evaluate the uncertainty in its momentum. if af = = Son, (p)=-infy eu = hafy tude = ha (p°)= rye Sa = (ia) [yt yde= Aa? Uncertainty in mometum (Ap) = o*)-(o)" =Vha'—ha <0 Example 26: A particle of mass ‘m’ is subjected to the potential enetgy 7 (x)=—fx?. Ata particular time, it has the wave function w(x) = Axe"*”/@”. Ifthe particle has a definite total mechanical enengy, find v the value of ‘a’, Soln. The operator correspéhdifig a +(x). Lethe particle has definite viaue ofthe totgl itecHanical ener Be iG elzenfimction of H, that is, 3 2 | iy L pygdgeatic eel a . +g kte Bat ele? { 240? 1 ma? mat 2 iat If Hy (x) hasto have the same functional formas y (x) one should not have the xe" /*" term. So, a4 240 1 (2 mk 12 are ular Soln. Concept of wave function Example 27, A particle moves in a linear potential V(x) =~be, (.e, under a constant force condition), Show that (x) follows rigorously the same time evolution as given by Newton's laws. ‘The equation for (x) is L-£\ m\ de =k Which is independent of ‘x’ and equals the force in classical physics. The expecta- a. Fz independent of x’, Thus 2.5 Schrodinge! Consider a particle time, The total ener Now, the 1-D time-depe —— i Heavmur com atr ek, VXT = ihX — 2m ot ine ‘Time dependent part: af =T(t)=Ce® ‘Therefore, the time dependent part of the solution ofthe 1-D Schrodinger equation is independent of V(x). Concept of wave function Soin. Time independent part: Therefore, the time independent part ofthe solution ofthe 1-D schrodinger equation depends on thenature of V(x). So, the solution of the 1-D time dependent Schrodinger equation canbe written as v(xt)=X(a)T()= x (xe Now, if v4 (x51), ¥2 (x,t)... (2,4) ate solutions of 1-D time dependent Schrodinger equation corre £ sponding to energies E,, Ey, Es,.--.E,, then the general solution is (xt) = (x54) +2 (x58) + eta Example 28: The wave fi i i Dpolehtialis y (x)= Aexp(-a°x? corresponding to eiger @ 26, (1-a2x’) 9) x ¢ ¢ Ei (1+207x*) Given: v (x) = Aexp| 2 > EY Aexp(-< Replacing all these the x Ta SaTEATOW a Bema | 2 WW: ree ANOUT.C | = = Aexp(-a*# Lgpreerendeay V(x) + Aexy )= EAexp(-a7x") Pgh? # Wa? _ na? = ~5 [4a 2a? |4V(x)2E = V(x) Spi ttt + te | Therefore, V(x) =2% + Fe 2kyats? | Example 29: The wave function of particle moving in a 1-D potential V(x) is w(x) = exp(~iax +b). The potential V(x) willbe the form: @V(z)ex ()V(x)ox ©V(x)=0 @V(x)ne™ i The Concept of wave function a Soln. Soln, Ee ry Ey ake +V(x)=E => V(x)=0 Correct option is (c). Example 30: The ground state (a part from normalization) ofa particle moving in a 1-D potential given by, x o(-S]oos (V2x). The potential V(x) is taking m=1, 2 =1) @= (0) -VBtani (Vx) () -VEtan( VE) @%—cotn( Bs} sinh (/2x) a LCABEE-ENDRANDUR | ESA sso SaaS > -El -1--yfypw Galeerendeavour.com Since, energy ofthe particle £ is independent of, then > ro-E(e ~2Ni tanh (V2x)}=0 = ¥(x)= Ee ~2v3 tanh(v2x)) Taking m=h=1 weget, => V(x) 77 v2 tanh (V2z) ‘Superposition of States: If|¢,),|9:).|4,)-.-|@,) are orthonormal state vectors/wave functions and are representing state of quantum ‘mechanics system, then any linear combination will also represent state of a quantum mechanics system, in other words if 4, mB, are solutions of Schrodinger equation, then linear combinationsuperposition of thestates Le. Concept of wave function Soln. Soin. |v)=C|4)+C.]4)+C,|6) +. willalso be a solution of Schrodinger equation. Note: _ (1) |v) will be normalized if |¢,/? +|C,/? +|c3| (2) Probability of finding a particle in |¢,) state = Example 31: Consider a system whose initial state at t= 0 is given in terms of 3 orthonormal state vectors |) I4s)s16) as 1 1 |v (20)) = F51A)+ 4|és)+ 7516) (Aisareal constant) (® Find the value of so that |y (x,0)) is normalized. in|), 4) 4) states. (@ What are probabilities regarding the system to be (Gi Find the state of the Site Example 32: The wavdhhbAW PALRRKARA BAY PULADM) where |) and |) arethe | normalized eigenstates with energies E, and E, respectively. The hamiltonian in the state y is E, ~2E, 2£, @ Be, Be, et oS 1 Suppose normalized wavefunction: |yy) = ¢[ 8 \#)+18) (where C is normalization constnat) 2 2 Normalizationcondition: (y|y)=1 => Chel acap ‘Therefore, the normalized wavefunction will be Ww)=-bia Flay (E)= P(t) Eo +P(A)E 2.6 Various Representation of wave function: ‘Wave fianction in position space is represented by y (x) ‘Wave function in momentum space or k-space is (p) = tal fv@ le _ Inverse relation: y (x) Ifa wave function is normalized in position space, it will be normalized in momentum space. But the normal- ization constant may be different. Example 33. Find the wave function in kspace for the wave packet, tion Concept of wave function Energy ofthe particle inthe given state willbe [ . thanpe 34 Fn WAANLGAFASFARCRA VOW GORE. v (x)~ ae!" : | Soln. 5 | -—t Pom P)_ [2% wns (yp — | = peg teal . ) FE as(0, p) i ‘Therefore, the momentum space wave function willbe 6(p) = AV2z1h 6 ( py ~ p) Example 35. Find the momentum space wave funetion corresponding to the position space wave function ofaparticle at a given instant y (x)= Acos x Soin. $(p)= J doostee de at Pa ¢ Soln. Soln, q Concept of wave function naff Bade Ha Sy dah[ 3 (nk p)+8(nk+ p)] =A Vala (nk p) +6 (nk +) + Probability Current densi As the time changes from t=0, the probability of finding the particle in some region of space rrayincrease or decrease. Ifthe probability increases in some region, then it should decrease in some other regionsuch that total probability of finding the particle in the entire space should be equal to one. We can assume this as a flow of probability from one region to another region, like a fluid or current, Therefore, the probability flow satisfies the equation ofcontinuty ie. VI+ ee 0 ot where p = probability density =y"y and J = probabil ‘The magnitude of pro : i s i.e number of particks passing through per unit area t is along the direction of flow of the particles. , (f= wave number, Bxample37, on MMNaLeSTENGeAIQUEGDI. (A) The probability current density for the real part ofthe wave fimnction y te ak @) OF Om @o (B) The energy of the particle is 42 242 22 2g @ Shtk 6) 3h°k ak ak om 2 a (A) The real part of the wave function =e +2 is 3cos kx ‘Therefore, the corresponding probability density willbe zero Correct answer is (d) a a wid Ps ix) | B 4 ket —2iK eo ® fy ( Sma et et 21K € i Concept of wave function Soln. Soln. Correct answer is (c) Example 38. Calculate the probability current density for a Gaussian wave pocket care “Ribchicon Example 40, Show that for any normalized wave packet in one dimesion { I(x)de=(p,)/m, J 4(@)ae “tire Lyas— dvs iva -afIralagper| where c.c. is the complex conjugate of the first term. Now.(p;) must be real, So, the first term in the square 1 1 bracket is — (Px) and hence its complex conjugate is =;(Px). Concept of wave functioiconc, [Flo] Example 41. The solution of the Schrodinger equation for a free particle of mass m in one dimension i wm [ewer ie WOst).At =0, y(x,0) = Aexp| =) Find the probability amplitude in momentum space at j= ane \ at ime 1 ¢ y Soln. 00-25 J wane de Q3. on 42 oof ve Vink ig V2h on ra) Soln. Qt (@) 2ih xp Soln. [x?,p?] ee “(aap Correct Optionis (). Q.2. Which ofthe following isa selfadjoint operator inthe spherical polar coordinate systema (r,0,9)? (CSIR June20121 gg ina a ino a pu eh ee ihsi Fin 6 00 OH © 36 (Hsin Og Soln, Anoperatoris said to be self-adjoint if [4° w de= [(44) y de v {i [00S \w(naoyasia sade oa =~inf i fe (1,09) ) (1, 8,9)r2dedo dp 06-060 inctio! Concept of wave function rension i, £=0 and Q3. Soin. Ta. On integrating by parts w.rt @ weget, = [ j 5G 2 ay wr? sinOdrdbdg sin 06 yr sinddrdddo : oye sin Bar d0do = Correct answeris (©). Given the usual canonical commutation relations, the commutator [A,B} of A=i(xpy~yp,) anc B=(yp, +2p, is: [CSIR Dee 2012] (@) A(xp.—pxz) ©) “A(P.-Px2) ©) A(xL+.7) A) ~A(H9, +947) A=i(xp, ~ (.B]=[i(o, yp) 8. =i[xp,. 99h eilyn.. Correct option is (c) Ifa particle is represented by the nostnalized wave function vis (ax?) lie eee V(X)=} ga 0 otherwise the uncertainty Ap in its momentumis ICSIR Dee 2012] vi0h Bn Oa 2m Sh OR 5, @ Concept: of wave function) - one xe E}e-x _ f Uncertainity in momentum Ap = ey) \=()" 7 a4) - VSh | Wea Correct option is (d) __ggostoncnsarrinnseserrammesnce Q5. Ifthe operators A and’ r@ 1 is the identity operator, then| t [CSIR June 2013]} 0-8 (@) [e\, B]= Al © [ec Bl= [eA] a Sol, [4,B]=1 & ef H Correct answer is (2) 6. IfA, B and C are non-zero Hermitian operators, which of the following relations rust be false? [CSIR Dee 2013] | (@) (A, BI=C () AB+BA=C = @ABA=C — @A¥B=C i Soln. Given: 4! = 4, Bt Cc C=[4,B]= AB-BA = (4B- Ba)! = Bat — ABT = Bd AB So, C=[A, B} is not hermitian. Correct answer is (a) Q7._ Letxandp denote, respectively the coordinate and momentum operators satisfying the canonical commutation relation [x, p)=i in natural units (fi= 1). Then the commutator [x,pe"?] is. [CSIR Dee 2014] (@) i= pye? (be) W-p*)e? © il-e?) © per soin, [x pe"? ]=[x,p]e? + pLxe?] Using the property [4,8C]=[4,8]C+B[4,C]) 2» rP tpt ZZ. weir ie? +p| _ [xp]+ hfe zn} Concept of wave function 2 ~i+ip-i 2! \ / *+plo- +7 2elnp] -t3p * [x, p] +. Correct Opti as ‘orrect Option is (4), ne 2013] 08. Suppose the Hamil constant and x andip ics is H =oxp, where @ isa corresponding Hamiltonian in ICSIR Dee 2014] _ iho 8 ae on 5amele abyaiod cna: opis ud duir.com 2 i | oa 2013), I | ! Concept of wave functi onc: ‘SET a 1. A.and 8 are two quantum mechanical operators. f[ A,B] stands for the commutator of A and Bl [4.8] [8.4] isequatto (GATE 2002] i @ ABAB-BABA —_(&) A(AB-BA)-B(BA-AB) (©) zero (@) [A.B] f 2. The commutator [x,P® ], where x and P are position and momentum operators respectively, is, [GATE 2003] i (@) 2inP (©) ~inp (©) 2inxP (@) -2inxP E io. 3. It[x,p]=ih, the value of [x’,p] is [GATE 2007] i (@) 2inx? (©) ~2ifx? (©) 3inx? @) ~3itx? 4. Three operators X,Y and satisfy the commutation relations 10. t The set of all possible gj [GATE 2007] | (a) {0,#1,42,43 @ jotti1a3, ' aes? AL 5. A and B represents: Hermitian, then for the produc AB to be Hermitian, itis IGATE 2009] (a) § is Hermitian ; 112, © Bis Hemitinang | 6. Which one ofthe fanctio the region 0< x <0, Wi iC egervAeT= [GATE 2009] 42" WWGATeereNdeAVOUL.COM, , cs. 7. Consider a quantum mechanical system with three linear operators 4, 8 and C which are related by where 7 is the unit operator. If 4= , then C must be (TIFR 2013} a nee 13, (a) zero Ong Oxy 8 The state |y) of a quantum mechanical system, in a certain basis, is represented by the column vector | lw)=| 0 VW “uncti t ' ad B , the related b) 1013] mn vector Concept of wave function The operator 4 corresponding to a dynamical variable 4, is given, in the same basis, by the matrix 111) A=|1 21 112 If, now, a measurement of the variable 4 is made on the system in the state |y), the probability that the result will be +1 is ITIFR 2013] ; w1 oF} @ + oR 05 ® | ‘The hermitian eonjugate of the operator(~2/éx) is [JEST 2013] (@) dex (©) ~d/ex © idlox @) -ia/ex r d The operator (eae is emuivalent to [JEST 2013] {¢ Palue of the product of uncertain [JEST 2014] @ Determinant (AB) ties A4AB is @ar Anormalized state nc) (oLGAREDR-SABEAUOUB, ee ee ‘aenuacmctoma serene ca cewentneeena [GATE 2001] @Xicr=1 www.careerendeavour.com (b) The average energy of the particle in the state V (x,t) is die, fe, () v(X,t) isaneigenfunction ofthe Hamiltonian of the particle. @ v(x, t) isaneigenfunction of the momentum operator, ‘Which of the following functions represents acceptable wave function ofa particle inthe range oo < x $ oo (GATE 2001] (@) o(x)=Atanx,A>0 (b) @(x) = Boos x,B real (©) @(x)=Cexp(-D/x*),C>0,D<0 — (d) @(x)=E xexp(-Fx’),B,F > 0 Concept of wave functio, 14, Anelectron ina time independent potential is ina state which is a superposition ofthe ground state aa (4-11 )and the st essed ae (£, =e) The wave faction ofthe dectronwilrepeat elt | with aperiod of (GATE2003, | (a) 3.11078 seo (b) 21x10" sec (c) 1.2107! sec (d) 1.010 see ' 16. Apatticle has the wave function y (x,t) = A[ exp (iar) cos(kx) ]. Which one ofthe following is correct? | I [GATE2003] (@) This is an eigen state of both energy and momentum, 24. (b) This is an eigen state of momentumand not energy (©) This is an eigen state of energy and not momentum, (@ This is not an eigen state ofenergy or momentum, ‘ Commom data for Q. 17 and Q.18 Ys. The normalized wave function y, and y, correspond to the ground state and the first excited state of patil ina potential. Youateivet the wave finctionsas Aiy, =, and Ay, =y, [GATE 2003] i 17, Theexpectation value he, (@)-0.32 ' 18. Which ofthe following (a) y, and y, i) heither y, and y, 19. Thetime independent hservation ofthe i [GATE 2004] (2) total binding energy Hef the system, i (©) total kinetic energy off | yn 20, Asystemina normalized site| AR BER of the system, requires that thé constants’; aiid ¢; iust satisfy the COhidition www.careerendeavour.com IGATE 2005] @elel-t — Wlelref=t © (elles) Ol 1 21. The parties of the wave functions (i) coskx and (il) tanh kx are IGATE2005} (a) odd, odd (b) even,even (©) odd, even (d) even, odd . 22, Ifthe probability that x lies between x and x+dx is p(x) dx = ae, where 0.< x <0o, a> 0, then the probability that x lies between x, and x(x; > x,) is {Gare 20 (ENE) alee) WEN (ENE) MERE -e™) 23, The wavefimetion of particle ina one-dimensional potential at time t = 0 is w(t=0)= Fel 2vs(x)-wi()] unctlo concept of wave function ) Citself 003) | } 28. state off, Ayrey, 003] | 26. ' | 004) lan. sigenstates 005) 05) 28. the rye ee where w, (x) and y, (x) are the ground and the first excited states of the particle with corresponding energies, E, and E,.The wavefunction of the particle at a time ‘t’ is: [GATE 2006] , 1 ei © eo 2el)-W 0] ) Jee [2¥0()-wi()] 1 ney 1 e ixvet® ogee vals) v 0] FaLvolne**-vinye™] A particle of mass m is represented by the wavefunction y (x)= Ae™ , where k is the wavevector andA is constant. The magnitude of the probability current density of the particle is [GATE 2006] ow mar wap oop ‘The wave function of particle, moving in a one~0. {GATE 2009] © yeneste® 0 @ r20 A particle of energy £ moves in one dimension under the influence of a potentials V(x). If F > V(x) for, some range of x, which of the following graphs can represent a bound state wave function of the particle!, (TIFR 2013} ‘37, @ © me vores SE AREER ENOEAVOUR) where 4 isa real Theexpectation velue of its momentuny [TIFR 2014] wt tel #) wun. pareerendegyaur.com., Atwo ste quantum system has two observables 4 and B. It is known that the observable A has eigenstates |q,) and |a,) with eigenvalues a, anda, respectively, while B has eigenstates |f,)and|A,) with eigenvalues b, and 6, respectively, and that these eigenstates are related by (TIFR 2013] |A)=Zla)=4}a).—(fr)=4la,)+ Zl) Suppose a measurement is made of the observable 4 and a value a, is ae Ifthe observable B is now measured, the probability ofobtaining the value b, will be (a) 0.80 (b) 0.64 (c) 0.60 (4) 0.36 inctlon concept of wave function agnituds.35, t 5 ma] | a state of 36. B) | Vex) for, » particle 13] 14] genstates 1») with 15) Bisnow B ‘The wave function ofa free particle in one dimension is givenby y (x) = Asin x+ Bsin3x. Then y(x)isan eigenstate of (JEST 2012] (2) the position operator (b) the Hamiltonian (©) the momentum operator (@) the parity operator The quantum state sin x|t)+exp(id)cosx|¥), where (t]|¥)=0and x,¢ are, real, is orthogonal to: [JEST 2012] in x{t) (b) cos x|t)+exp(id)sin.x| +) (©) ~cos x] t)-exp(i9)sin x|4) (@) ~exp(ié)c0s.x|1) +sin x|1) Ifthe expectation value of the momentum is (p) for the wavefunction y(x), then the expectation value of momentum forthe wavefunction e™*y (x) is {SEST 2013) @k () ‘The Hamiltonian o tates corresponding tothe larger i [JEST 2014) (v2 +1}|2) 3 +1))1)+|2) € g is a constant) and respective @ |) -(2+1)l2 © |1)+(v2-1))2) Consider a three-state ay. eigenstates "Hthesysemis alvin Déaneerendeavour.com lwi=|o ° What isthe probability that ata later time "system willbe in state 0 vs)=}o (JEST 2014] L 3gt gt E-3gt) @o 5 Asin’ (#) oF feos 38) ( dsin ( ie ] Concept of wave functior, “Aall bounces back off earth, You are asked to solve this quantum mechanically assuming the earth i a infinitely hard sphere. Consider surface of earth asthe origin implying V (0) =o anda linear potential eleswher, (ie. V (x)= -mgx for x > 0). Which of the following wave function is physically admissible for this problen, (with k> 0): (JEST 2014] = 2 @ y (x)= Axe™ @ v(x) ke = Ovex" Ov( Axe! AREER END! Se HET www.careerendeavour.com ‘unetion earth is af One-dimensional potentials 3.1 Introduction to Bound States : | Ifthe motion of particle is confined to a limited region of space by potential energy so that theparticle can ' move beck and forth in the region, then the particle isin a bound state. The simplest example of ll motions in bound state is the motion of a particle in 2 one-dimensional box with zero potential energy and the potential energy is assumed to be infinite atthe walls ofthe box. ‘The motion of a paytioles ig x oscillator, motion of at x, motion of a simple harmonic ith (E < Y,), motion of a particle cle in one-dimensional (also in potential, ete. are the common equal to such problems leads to of discrete energy and non-degen- (i) The wavefunct ric potential should have adefnite Parity ie. it will (i) The wavefuncti responds to the ground state of the particle and Will ‘ state of the particles. 3.2 One-dimensiofip iliely ech Palatal Wal Consider a particle of mass ‘mand e ving along X-axis, inthe region fromx = 0 tox =a under *eeswtementivw.garegrendeavour.com = otherwise Veo Vaw ve v=0 x50 xéa — Figure : Schematic diagram of 1-D infinite potential well, ‘The particle experiences no force within the potential well but feels a sudden large force directedtowards the origin as it reaches the points x = 0, a. Since, the potential is infinite outside, the particle cannot penetrate outside of the region 0. &, =. 242 Firstexcitedstate energy: n=2. => E, = 27 mal Ingeneral, E, =, Difference between} Spacing between t particle ‘m’.As a $4 GEE aotrhing a continuum of energy. PAREER ENDEAMOU z as = witere's* vac aummutateanantleaveus coms.” Difference of momentum between two consecutive energy levels fe My Ronal Eten ont ga a a a Various eigenstates of the particle inside a 1-D box: i vie) a en Ground state First excited state One Dimensional Fotentiaj One t | i | Note: y, (x) has (2-1) nodes (excluding the boundary atx = 0 and x = a) ‘Variation of probability density corresponding to various eigenstates of the particle: Probability density of finding the particle in the n® quantum state is given by, 2, (2) =n (f= Zsa? el oad bes | Figure : Variation oy ‘density corresponding to ground, first excited and second excited state respectively Note: Stinging the particle inthe eta i: | ' orthonormality condition 4 {- i : : (x) (whichis well behaved and ‘igenfunctions as j i i Soln, « ( (=fv(-02) yde=—(-ih) 2am fs) 7 cos (lao oe Note: Expectation value of momentum for a real wave functions always zero, (#)= Ive vara 2h sin? de eee ireer eal qe ae oe 2(nay¥ | 3 2nn* (e=2fan( 22) & (2 )acm2( (82) Canf (Ma Example 1: apes ina rae ATS ‘extending from Sr w.careerendeavour.com v(x lla) ) +2|4>) | where |) and |p) denote the ground state and the first excited state wave functions. Find the expectation of x” inthis state. sam. (*)=(vle\y)=fe((Al 20h) Fella) +214) = ELIA) *2( ble) +2 (dbo) +4(62Iol¢)] One Dimensional Potential Soln, Soln, Soln, 32h gL) 6ab ont 2) 2 45a? ue 32L Therefore, (*)= 5] 5 ~ 9 Example 2: Calculate (p,) for ¥ (*) alla) where |g) and |) are the ground state and the first excited state wave functions of a particle in a deep square well potential. 4 ein 27E oye EH J2sin=* cos de aL \f “x” in this state, The 25 nD Example4: Apartice fs x= ofthe wellis shifted to’ 292 rT. yi range 0 > |G,=1 hts #Soae ff 5 Ve One Dimeneional Potential fe, E fs Therefore, |v (x.0))= ysl) yeQ lO) yagi) (b) The state of the particle isa linear combination of ground state |g,) , 2nd excited state |g,) and4th excited state |@,) respectively. Therefore, the energies we will get are on 2ma? "> 2ma? ‘Now, the respective probabilties are 2 3 ~ e_ 3 PE)eKaiwil =z P()=liv Pig PC (£) = P(E;)-B, + P(Es).B + P(Es)-Es 3 Ont Sah ~3°2ma* "10 2ma® "10 2ma? we a 27.5} 2908 “ama? (5 °10"21" 10ma® © W9) = Sime eA “fle ® bao)elaneeh ou a“ = 0 Example 7: A particle of mass m confined to move in a'poténtial VQ) 0 for 0 4 P| (asin sat) d=) a av a @ : ) Therefore, W(%0) 2, Pants 2 soe |g (2y, +5) Vara Gi) VOD) aygeves +ye"") val | One Dimensional Potential | Example 8: A particle in a box is in a superposition state and is described by the wave function ~a me Y Vays = Ey; > 2m(V,~E. : Aasuming «= = 2mo-B) and p? = 2 Region-1; an -ay, =0 Region-2: Me + By, (Sb) and (Sc) can be written as (5b) (Se) (6a) I | | | | i | | | | (6) 19 ma, =0to (6a) (6s) One Dimensional Potential ) Regions: 4 Sey =! (Ge) Solution ae (62), (6b) and (6c) will be ofthe following form: y a <2 Vile de + Be («5 ) 7 (x)= Coos Bx+ Dein Bx |~ fs x< v Wi)=F ese x28) Boundary conditions: @y (x) > 0atxoa0 oy v(x) ona Hare continuous at x= Using the fst condition, we get B=0 and G=0. Since the particle is moving under a symmetric potential, then the wavefunction of the particle should have a definite parity i. eth (2) Even bound staf ‘The wave function df Now, boundary cont ops ® Cain BS (10) ovstgeqanec sategiendeavour. com a) Weknowthet, a? +p? = ae a Defining PS = pandas =9,weget pram p = q and p? +g? = 03) Ira ‘The second equation represents a circle in the p-q plane with radius R = (ae which depends upon the the height M, and width q ofthe finite potential well. Both of these canbe solved graphically and the points of 3 mnt eee One Dimensional Potential Pr = Pos P2»Par- which also gives the discrote spectrum ofenergy valu for the even eigenstates as (1 =0,2,4 6.00000) pager’ ‘Number of even bound states: 0 (w#1)even (2) Odd bound states: oe ‘The wave function of the particle will be of the form: ~ wila)= er" Now, boundary conditi «sy ihe dS vt | Wl re) dy a "5 ADeosp 2 Malye 28) Wal p22) ae? =pDcosp* (16) | fib cated ondedveur con > i i Dividing equation (10) by (9), we get, pox BS =-a an | i 2mV, We know that, op aae 18) Defining pS=p and a q,we get | ha? poot p= —qand p? +g? = Meet! (19) 2h? Both of these can be solved graphically and the points of intersection of the curves] 2 gand p+ 4 ives different p values ie. nia’ He (1s) (16) ay ca} yy curve One Dimensional Potential ‘hich also gives the discrete spectrum of energy values forthe even eigenstales as apt PAPE (1 =1,3,5.Tonens) ma’ eotp=4 ‘Number of Odd bound states: is <= = one odd bound state Example 9: There are only 3 bound states for a particle of nmass ‘m’ ina 1-D potential well of the form: (x20 = S| One Dimensional Potential Soin, Soln, Sol. The depth of the potential wel satisfies the condition. anh? ont 2a ann S othe first branch does not exists. 2 “a Ly, The wave function fora bound state at energy E can be written as, (solving 1-D time independent Schrodinger equation) 4 72 ERB a Inthe first excited state, the wave function is ofthe form y=-Asiné . The maximum value of the wave tnctonisa cork CARRERE NGC OL FROIN, N a7 La an 5) A si Bd “+ 2L 2 kL Ax Asin ins 2n Example 14.A finite square well potential has length Land depth V,, A particle of ass 5, 7 is trapped in ‘ this potential. Show that it has only one bound state and the energy corresponding to this states satisfies cos((E7%) = EM% One Dimensional Potential 2 LY [2mr, Soin, p> +a (3) [20 an VoL? | Moe wy on [a Thus, VP” +4? <5 and hence only one bound state ocouts. Using, p= BE mine rd woof 3.4 One-dimensional step potential: Let a particle of mass mand kinetic energy # is travelling Pen potential step at = 0, given as following: Va)= { 22 to the x-axis and is incident on the 0 forx<0 Vo fore 20 The energy E of the — From classical point of vie will be reflected back-at be reflected back at x = 0 iaste Quantum mecharcally wil be shown that theletron willbe pry ¥feted and party transite a X= 0.This so TARR ACEH ETON GEA VOUT.COM Case A: (E> ¥,) ‘The time-independent Schrodinger equation for 1 dy, dy, oN. i Region 1: 75TH =Ey, = ah (20) Pa Py 2 Region 2: oe Ey, » Ee Voy =0 v2=0 (21) mls 2m(E =F, where, @ = ie and B= aa ‘The general solutions of equations ate atax «mix >0. ‘cle not dat 20) (21) ny | | One Dimensional Potential ‘The terms e!* and iar represent respectively a wave travelling along the positive x-direction ie. incident wave and a wave travelling along the negative x-direction, ie., a reflected wave. Again, the terms ¢f* and e-® sepresent respectively a wave travelling along the positive x-directic ‘wave travelling along the negative x-direction (a reflected wave). But due to the presence of only one discontinuity at x = 0, there cannot be any reflected waves in region 11. Hence D = 0. Therefore, Applying the following boundary conditions i.e transmitted wave and a (dy, dy; © Weo=Wano and Ci) (2), . wewillge, B=222.4 and c={ 2% ]4 B a+B therefore, wavefuunction in region 1 and region 2 will be (23) oe 1p Tava GaneorUt clawoal CONF Jytde= 47 Cee =p, 4ap f= a we I td,4, “+p (ab) ice. total number of particles is conserved in the process. Reflection and Transmission Coefficients: Reflection coefficient e a2 _ Reflected ftux(,) m2! jah (a-B) _(vE-JE-% Incidentfiux(J,) Rey 42 [AP (a+B. One Dimensional Potential ‘Transmission coefficient a Transmitted flux (J,) Bick Bich 40p_ __ 4JE(E-%) Incident flux(J;) AG ajar (a+py (JE = AP (VE E+ JE~ %) Gi) For E=Vy,R=land £>>%,R=0 (iii) For £=%, 7 =Oand E >> Vy, T = Gv) Amplitade (C) of the transmitted wave is gteater than that of the incident wave (4) as ct > B. (v) De Broglie wavelength in region I will be shorter than that in region IL. This is due to the fact that the kinetic ‘energy of the particle in region [is greater than the Kinetic energy is region II. The nature of the wave forms is shown in figure, z z ability of being reflected and ‘The general solutions of equations are y= del + Be, yy CoP + Det (26) But the term, e* — co a8 x~300 which contradicts the condition of well-behaved wave function, ‘Therefore, C = 0. al de te One Dimensional Potential Applying the following boundary conditions: a (4) -{ v2 OWDeo=Waco ond He) ae), f4B 4 and p=( Z| 4 ia—p ia-B therefore, wavefunction in region | and region 2 will be = sox . (7048), ~iox wode (=e we will get, B= (27) x0 a ‘The phenomenon is very much similar to the case of total internal reflection of electromagnetic waves while travelling from denser to a rarer medium. Quantam mechanically, the wave function of the particle in the region 2 is non-zero. therefore, the probability of finding the particle in classically forbidden region (region 2) is A(x) =Iyo(a)f =pr ee ‘Such penetration of a barrier by a particle (having energy lower than barrier height), falls exponectially with distance from the step Dotential barrier. Soln, One Dimensional Potential Penetration depth: It is defined as the distance at which wave function reduces to I/e of the value of the wave function a the barrier (x = 0) and given by 5 ate ‘The fact that particle can penetrate into the classically forbidden region leads to Quantum Mechanical ‘Tunnelling. Ifthe potential barrier is infinitely high Le. Vy — oo, then penetration depth becomes very much negligible Example 15. Electrons of energy 3.8 eV are subjected to.a potemial = 0 for x <0 and = 7.6 eV foc x> 0. (a) Find the wavelength of the sinusoidal vatation of the Wave fumction in the classically alowed region (b) the reflected wave differs from the incident wave by a phase factor. Find this phase factor. WW. Gareerendwavour.cormn, a a < |—— 10am 38x10 e7 nm’ (a) The wave number *k? The wavelengthis 4 = 2% = 628 a nn (©) The wave function in the classically allowed region is = 0,63 nm a4 ear, 1atB iar vale tbe] Thus the reflected wave differs from the incident wave by a phase factor Webave a =10.nm"' and similarly B the sical uch, One Dimensional Potential Soln. Soln. Example 16, Electrons of energy 10 eV are incident on a potential step of height 13.8 €V. Find the distance in which the probability density of finding the particle decreases to a factor of 0.01 as it penetrates into the classically forbidden region. “The wave function in the classically forbidden region is ofthe form ce According to the question, P(x+d)=0.1P(x)=|cf &"" =0.01\C) ia => e474 =100=> 2yd = In(100) => d bit a, 2B3nm Y Example 17, An electron beamis incident on a potential barrier of height 0.03 eV and ofinfinite width. Find the reflection coefficient and transmission coeflicient at the barrier ifthe energy ofthe incidentelectron beam be (a) 0.04 eV, (b) 0.025 eV. (@) In this case, E> Vy Reflection coefficient (ee ‘Transmission coefficient = (b) In this case, E < Vy. reflected back and no ¢lectrot R=1-0.1=0.9 T=Oand R*1,,i¢. all the electrons that are incident on the barrier are ‘will be transmitted through the barrier. 3.5 One-Dimensional Rectangular Potential Barrier Consider asweam of particles each fis sm’ and energy E be incidént from let on the potential barrier surface at x = 0; given as Ho The following two cases arise: Case 1: If E> V,, then according to classical mechanics the particles will be wholly transmitted and no reflection is possible, But quantum mechanically there is always some probability of reflection at x= 0, back in segion I Case 2: IfE < Vy then classically the particles will be wholly reflected and hence penetration through the barrier is impossible, But quantum mechanically there is always some probability of penetration into the barrier and appearance of the patticks in region IIL. This finite probability of transmission through the barrier even EV, is called the quantum mechanical tunnelling effect. One Dimensional Potential Case A: When E> V, ‘The time-independent Schrodinger equation for dy ayy, 2mE PU ody, = Region 1: ee By = Pe Tra p=0 => Fa 28) # ava ayy 2m OVE Rey a Region 2: ~—— aia +HW2 = Ev, => we +57 E-Vov2 =0 en +B'y2=0 (29) Pays yy, 2mb Puy Region 3: -5——> mae =EV; => ea ps0 => ~ ht ys =0 (G0) where, 0? = = and p?=2mE =o) | ¥ # ‘The general solutions of equations are ES + Geri Since there i no refi Applying the allowing, eB) es =f GHOBAMOUR won ih SABEER OD ms Substituting the “aN G@NREIENYIGAVOUECOM rs ée[-at I] 2 “[p-Bpaglew B-g\"] oy A= From equation (33), bial 28) 29) 30) sy i4) One Dimensional Potential Fo Soper‘ 2ape i o 4 @s By(eF*—e) -2ap(eP +e) 2aPeospa— ia +P)sinba Reflection and Transmission Coefficients: Reflection coefficient hae Reflectedtux(J,) _g !2P_ ype p= Reflected fu (J) _ m1 BI A hs (il As B increses al ‘ rapid compared vo sin? Ba, for i a SsameatOoan... wef he pari in region M1, then there will be perfect transmission of the incident beam of particles. This is Ramsauer- Townsend Effect. (v) For E >> Vy, T= 1 and R= 0. The natures of the wave functions y,,W> and y3 in region I, II and III respectively are shown in figure. Region Region it Reon tt “ampli Case B : When E < V, The time-independent Schrodinger equation for Cie e wil A=S {1 we will get, <( i Applying the folowing! 78 @ Wr) ena = we will get, 3 > (tg Substituting the sone WAV GAEEGLENAAANQUE.COTax set, Pala i ape" A 2afcoshBa+i(B? — a)sinh Ba q@ One Dimensional Potent (40) tial G6) e7) G8) 9) (40) a) (42) One Dimensional Potential Reflection and Transmission Coefficients: Reflection coefficient _ Reflected ux (J) _ “in wise _ [BP Incident flux (J;) cp TTAP m 24.92)? (eA sinh? Ba 40°" =R= 2 7 fee ane 1 vo inh? H{ “t) sinh’ Ba *4EW)- 5)" be ) ‘Transmission coefficient Tease fx) 7 — = 4a" cost + int ba 4EVo-E) Nate: ( There isa finite jal barrier of width a and height V, even if E < Vy, (ii) As E increases, ( | The rate of decrease of sin” Ba for a fixed value ofida fase steadily with E. | a 2 i [reste ies iwwrcat6erenieavour.com 16E(%-E) W sinh?(Ba) Therefore, T= ze The natures of the wave functions y;,W2 and y3 in region I, I] and II respectively are shown in figure. qe One Dimensional Potential Soln.: Soin, Bev, E-V, E>V. E=>v, ES Example 20: Calculate the percentage transmission of an electron beam of energy 3¢V when incident on a potential barrier of height 4eV and width 20A. E Transmission coefficient 7-16 (- ze Yo What is the probability that i will escape from t tthe surface of the micleus is 4 MeV and the mass! an ww career endeavour. com Given: E = 2MeV,V,=4 MeV,a=2x1.4 x10" ‘Therefore, transmission co-efficient will be T = 0.124 3.6 One-dimensional linear harmonic oscillator: Ifa system vibrates about an equilibrium position such that (a) the magnitude of the force acting on itis proportional to displacement from the equilibrium position, (b) the direction ofthe force is opposite to that of displacement, then tis called linear harmonic oscillator (LHO). A simple pendulum, a spring -block system, a floating body etc are familiar examples of a dynamical system that perform simple harmonic motion. In quantum mechanics, linear harmonic oscillator is used in place of simple harmonic oscillator. Classical Linear Harmonie Oscillator: Leta time ‘t, the displacement of particle of mass ‘m’ constrained to move along the x-axis be x= 0 fiom equilbirum position. Restoring force on the particle is j° = —fx (kis constant). aton y that | us is One Dimensional Potential a re ax =k => —F4@x=0 e na qe tO =0 (where of ‘The solution of equation (1) can be written as x=asin(or +) This represent the simple harmonic motion (SHM) of the particle with frequency, 2x 2x ‘Therefore, potential energy of the particle E, and Kinetic energy of he particle apm Quantum Analysis, A particle of mass‘ aN EOI} fy ay ay 2m £4.28) phn y 0 Co Solving equatian (42), we get the normalized wave function oflinear harmonic oscillator in the f* state is 2 a meta ame = H, (ax) = v, (x) (x r| é (ax) where, & "i (43) First few wave function of the LHO are as follows: ‘Ground state (n=0): One Dimensional Potential va First excited state (n=l): wbl=(s3e) (2ax)e**? Second excited state (n=2) vie) -(%5] (4x%a? -2)ere?? Third excited state (n=3): yaa) Note: (i) y,, (x) is anf Gi), () has Energy eigenvalue of 4° excited state 3 excited state 2" excited state 1" excited state Ground state On: (Solr tial Zal | o oimensional Poveneiat &S + Existance ofero point energy according to Uncertainity principle: Let a particle of mass ‘m’ executes SHM along x axis and at time ‘t’ its position is x from the mean. position. Total energy of LHO E = E, +E, =2—+~. AL A Ax x the = Since, Ax= x then Ap = 5° = 5 2 Therefore, E=L( 2) Qm\ 2x) Paine F = 9 = where y,and y, a potoreapcaively The expectation value of eng 31 io" win @areerenlaelfour. cont? 1 ia y (x) should be zero at x =0 ‘The normalized wavefunction of a particle moving under a linear harmonic oscillator potential is v, (x)= Ae? 7H, (ax) [view na ¥,{X) willbe non-zero forn=0,2, 4,6, une and will be zero for n= 1,3, 5,7, So, possible values of 'n’ are odd n= 1, 3, 5,6, () The energy eigenvalue of the particle willbe eyx(2ne ted} om (and (Gi) The expectation value ofenergy will be (6) = PE, + PE = dS hos dc dnme am One Dimensional Potential] Soin. Soln, Example 25: Ina 1-D harmonic Ostillator, two states are defined by = 26, +34 Va= hhh rae, where ¢,, 4,4 arenonulized wavefuunction corrsespondingh to ground state, first excited state, second stat respectively, (@) Thevahe of a forwhich y, is orthogonal to y, is (@2 1 @-1 (d)-2 (ii) The expectation value of the oscillator in the state y, is @yhe She este é, Lia ‘Example 27: Aparticl 3r potential V (x) = yh tthe spring constant is sud ticle in the groundstate of ney} potential Normalized ground: Iwrivllef. (= rn ae OM yt Fe a @avour.com Hamiltonian ofthe system is Defining two dimensionless operators paoh and X=$,/"" Ineo ntial One Dimensional Potential ad state » afta of new} The hamiltonian can be written as, HT = Introducing two non-hermitian operators, @=: plist, where @ and at are hermitian conjugate to each other. Now, aa =3[ £2 +P? +8 102] = ata fis As -(#art Sho >A= (w+) Dro where, N= atq = number operator/occupation number operatat. + Some important commu olaa]=1 fi ( [A,@a}=0 a i So, that, dn) is the eig & (Ea), Actionof 6 @ ( on 2) gonerate ney rat (ol So, alm vi dareerendeayour.com Simily, 1 (4"|n)) = (414 + heat) =(E, +he)(@*|n)) So, that, a" |1) arethe eigenstates of AF operator corresponding eigenvalue (£,, +h). Action of at on|n) generate new energy states that is higher by one unit energy Aa. at |n)=e,|n+1) > (n]a= (n+ lle, Therere, (naan) fey n+l] +1) = (n+1) fey lel = VFL So, at |n) = Jn4T|n41) ‘Therefore, the nth state can be written in terms of ground state as = invaltD ins One Dimensional Potential Calculation of normalized wave funetion: Ground state wavefunction: Since, action of @ on |7) generate new energy states that is lower by one unit energy fr , then alo) =0 fees [ma h 35 mea =0 = [me ay, +t ny, = V2 a eal ie Van" aman ° = ayy <9 eee te = me ong a oye Won = ¥o(0)=Adem[ MEE) ysle)=Aer[ 8 Normalization Condition: {5 (x) First excited state ws Since, at |0)=\1) ateerondeg sept 2 ats? a e 2 A(ax) Expectation value of a odd operator for'a symmetric potential: ‘The wave fimetion ofa particle is moving under a symmetric potential V (~x) = V (x), has a definite parity ie it wil be ofeither even or odd parity. Therefore, expectation value ofan odd operator = (4) =(y|4ly) =0 Similarly, 4") = (y| 4” |y)= 0 (nt = odd integer) Soln, al Soln, one Dimensional Potential @ ‘Calculation of expectation value using operator formalism: gu lmes [re Asticknow, d= 8 + Pome ang a= therefore we get, tint) ns af PPA) = ohare Je) =e Lvaten- y+ VnrT(ninsty]=0 le (a-ha A 1 teint) (nine) 7 =alnlla ea” vat +4'a)/n) Shing {n|n)+ Vn (n\n) Example 26: Let lola demreecrertckteres stom to the ground and first state ‘ofa L-D harmonic oscillator. The uncertainty Ap inthe state a (|0)+11)) is: oak nah oe o (# (© dion @ Vinh (=A) (aha) (p= o+1 AE tA 20x Co) =AL Ooh (oo) lpo)-Garp]+4[mind+0+0¢3mto] «mon ‘Uncertainity in the momentum in the given state will be dp=le\=(0 = Vimo One Dimensional Potentla, Example 27: The wave function of. linear harmonic oscillator in the potential V (x) = Aye a Bye Ha! = Agel i) ayn) _ dv hol _p erital) — ip 4 eth eae ke = ky (Agel — yea!) = hy Ag’ (i) ‘According to the question, git2l . ei 0 where 8(x) is the Dirac delta function. Find the energy of the system. Sols. ‘The Schrodinger equation for such a potential is _# Py) 2m dx’ 7 V8) W(x) = E(x) ntial lensity pees Se Since the potemial is attractive, then £ <0 for bound states ao For all points except at x= 0, the equation becomes Soloution of the above equation, will be e*, x>0 wor fer x20 Q) (the solution must vanish at x -> +0) The normalization factor is assumed to be unity. Integrating equation (1) from — to +2, being an arbitrarily small positive number, we get Example 35. Cons nstant k. The interaction potential is H = Ayx, wher § Obtain the energy eigenvalues. Soln. The Hamiltonian veil Sat mot tyes fone 2m av: Hence the system can be regarded as two independent hatmonic oscillators having coordinates y, and y,. ‘The energy levels are Soln. Soln, CSIR PREVIOUS YEAR QUESTIONS 1 The wavefunction ofa particle is givenby V -(e +) tee Go and ¢, arethe normalized eigenfiictions ‘with evergies E, and £, corresponding to the ground state and first excited state, respectively. The expectation value of the Hamiltonian in the state y is: {CSIR June -2011) Ey Ey : Eo~2E, Ey +2E, @ yr pF @o“5> ary Normalization of y: w)=4l ia) a)] 2 Applying normalization conditionie. (y Iv) =1 ots a1 Correct option is (d) | A particle is confined ly a5 u(x) =tlgx . The mean position ofthe particle R June-2011] k,T ay According to virial The 5 (Potential energy) For given potential u(x) HS RUE BEL) = aia SHE) according ocounans ff W.GareeTendavour.com (Energy) = (KE) +(P-E. uy (x)= kel (poe Correct option is (a) Consider a praticke ina one dimensional potential that satisfies V(x) = V(x). Let yo) and | y,) denote the ground and the first excited states, respectively, and let | y) = c.g | y,) +0, |) beaaormalized state with 1, andi, being real constants. The expectation value (x) ofthe position operator in the state |y) is given by [CSIR Dee 2011] Sc ial mms jon ny note state ly) i) One Dimensional Potential soln. (a) 5 ¢Wo 1X] yo) tary, [x1 Y,) (b) 1,04, [yo X19, OY, 1X} Yo)] (©) aj tor (@) arya, (y= WHELY) (yas +(vals)x(aelys)+04/¥)) (ly) = 5 (Wo x{ yo) +03 (ys|x]¥.) + toch (Yo [xl yi) + ac (will Yo) For a particle moving under a symmetric potential, the wavefunetion will have definte parity Le. ether even or odd, Therefore, the first and second term will be zero as the corresponding integral will be odd innature. So, (&) Saf (volal¥:) +(wib¥6)] Correet option is (b) Let | 0) and |1) denote the normalized eigenstates corresponding to the ground and first excited state ofa is ICSIR Dee 2011} = ele +a -2aa-1] a | lar left) + (a (=O ~2(01ata} +(1faalt) + (ofa’aa) +-(als*afo) —((010)-+(111))-+(o11)+(110)) 0) + (ola"]1)+ (1 lo) + (171) + (ofa 0) +(y 1) +(0 ) ame) POM == Ame = Ap = Vimo Correct answer is (c). One Dimensional Potential 5. The wavefunction ofa particle at t= 0 is given by |y (0)) = glu) 4+fug)]}, where |u,) and |t42) are the normalized eigenstates with eigenvalues £, and E, respectively, (, > E,). The shortest time after which |v (2)) will become orthogonal to |y(0)) is [CSIR Dee 2011] hi 2h 2hn © @-B © @-£) -E) Soln, ‘Therefore, the minimuh| a Correct answer is (b} 6 Aparticle in one-ci 0 If there isat least one bY AAR) wre Re 20? wre sme Ome One Orne wo if x<0 Vy if Oe Son. V(x, Region: y; 2 Region I: —" es Ve)¥u =O dx? reerendcavouneem [CSIR June 2012} ° So one Dimensional Potential iS tial Solution: yy =Asink,x+Beosk,x are Pq, 2mE &y, 2 _—2mE iM - iB igy = ei = mE a Region IM: Gah + Wg =O SEE Kiar =0, where Ki = Soln, Solution: yy =Ce** +De“** Since, the wavefunction should be finite everywhere in space, therefore C=O and y,, = De“** Boundary conditions: () Wi hoo Vu h-o=0- Therefore, yy = Asink,x GD) Yuko Yankee = Asink,e = De® ay! Wy ii) —4 => kAcosk, a Using the equations from conditions (ii) and (ii), we get =k, = k, cot (kl) = kf =~kl cot (kl) > B ‘acota _ _suvneeayocontdeayoyrcti Correct answeris (a) 1 The minimum energy of collection of 6 non-interacting electrons of spin ~ > placed in a one dimensional infinite square well potential of width L is [CSIR Dee 2012] @ i4s2n? /mi? (0) 91? mL? (©) 7A / md? (@) 397? / ml? ‘The energy of a particle of mass ‘m’ in a one-dimension square well potential of width L is given by ath? boo ea 2m 1 Only two electron with spin = can fill in the same state Soln. (corresponding to minimum energy) The minimum energy of the systemis, We ann 2ml? 2m? Lah ml? One! Correct option is (a) 1 ‘The energies inthe ground state and first excited state ofa particle of mass M = > in a potential V(x) are—4 and—1, an Gini sinsvbichsp. wy (x) =Wo(x)sinh %3 @ valshexa ‘The time independent jelfthecomssponding wnseetins are elated by cr [CSIR Dec 2012) 9 (X) =sech? x For ground state, Agyak tae Fortiexced sat pa aeons mene com Since, y,(x)= yo (x)sinhx s [-gs+v “fy o(s)sinh x=—y, (x)sinh x = SE (vels)sinnx)+ V(x) Wp (x)sinh x =—y, (x)sinh x ix = ~ va(s)coshae sis Me x ih. a o(x)sinb x }-vGa)vesinns= & ; = ~We(x)sinhx- ~cosh xe ~coshx te. sinh x- SEV Oy (x)sinh x =~, (x)sinh x One Dimensional Potential oO Se tial re-4 012] inhx One Dimensional Potential Soin, dy, = Boosh Ste ~sinh x F%2-+ V(x) vo(x)sinhx =0 Pug, 2coshx dy, _ a sins de OMG) Using the Schrodinger equation for ground state, we get > = Me, Zeoshx due ave gy, () ax sinhx dx de dy, __» sith x dy, __2sinhx Wo 2 => Me. Gx eos V0) yy cosh => ny, =insech?x+InC Therefore, the ground state eigenfunction willbe y,(x)=Csech?x Correct option is (¢) Let |0) and |1) denos found and the first excited states of one-dimensional ((o)+[1)) [CSIR Dec 2012} @ Ax=Vh/2ma) @) Ax =Vh/4mo 1 )=(vlely) = Zt ; ’ deay ULC e( Eye nt ai rr om (0) = ss HCl) + (0b) + (ao) (1s) obo) (et) +o) Ger) +2((0|ata o}+(1fa"a) + (ofelt) + (1faa}0)) + ((010)+(111)+ (02) +(110)) revs 3:= (EO = fo Correct option is (a) Soln. One Dimensional Potential Consider the normalized state |y) of a particle in 2 one-dimensional harmonic oscillator: (yw) =b,[0)+b,|1) where |0) and |1) denote the ground and first excited states respectively, and b, and bb, are real constants, The expectation value of the displacement x inthe state |y) will be a minimum when [CSIR June 2013] (2) b= by 7 =) ORB Ob Given : Normalized state }y) of particle in a one-dimensional harmonic oscillator is |v) = by|0)+bo|I) ‘Normalization condition : b? +b3 =1 Here |0) — ground state |) + fitst excited state Expectation value of = b? <0|x|0> Since, x is an odd fu ion will be zero. Pa — : bby [< CAREER. ENDEAVOUR cee cece eras Therefor, so= pr Cae brebv Brom So, b,b, will be minimum if b, = 0 or b, = 0 Casel: = b =O=>b)=1 Case 2: b) =0=>b) =1 [Due to normalization condition b? +b3 =1] ‘So, correct option should be (@) In the answer key correct option is given to be (d) i.e. b, = b, Ifb, =b, => b,b, will be maximum and its maximum value will be b? or b3 then, will be : : i z maximum and its maximum value willbe 267 |" or 205 />—— Correct answer Is (a) So Sol One Dimensional Potential it. Aparticle is in the ground state of an infinite square well potential given by, 0 for-a E,.At time ¢ = 0 the particle is prepared ina state (+ =0) 1 +q)- The Yr (Yi +4) shortest time Tat which ¥(¢ = 7) will be orthogonal to ¥(¢=0) is [CSIR Dec 2014) 2h @,-F) @) Soln. . Given: At t= 0, the The wave function of Since, y(¢=T) and a earl oer LNW. v cafgerendeayour com eth, vasa wo elBBIP yc gfe = Ar (Qnti)a T= Caciet For shortest time taking n =0, T= Correct Option is (b) On 4 Sol intial One Dimensional Potential nd E, )-The 2014 14 Soln. Let |v) =c,|0)+¢,|1) (where c, and c, are constants with c3 +c? =1) be a linear combination of the ‘wavefunctions of the ground and first excited states of the one-dimensional harmonic oscillator. For what value of c, is the expectation value (x) a maximum? [CSIR Dec 2014} al | a oe I h 1 t h 1 © 0)" Vina "5 OO) Ynar 5 oe a E Given: |y’) = ¢9|0) +¢,|1) with gl? +leif = Expectation value of position = ‘The maximum valueof Correct Option is it ee www.careerendeavour.com One Dimensional Potential PRACTICE SET 2m Ifthe wave fimetion of a patticle trapped in space between x=0 and x = L is given by W(x) = Asi( 8 rE ), where Aisa constant, for which value(s) of xwill the probability of finding the particle be the maximum? [GATE 2002] L L L L L 3L ie . Eng E © and = OF OF OEMs @ za = The energy density of states of an electron in a one dimensional potential well of infinitely high wallsis (the ‘symbols have their usual meaning) [GATE 2003] @) Lim [ah (28)| (b) Lm/(nh VE) (@ Lm/[n/(2E)] (@ Lim /(2n AE) A free particle with energy E whose wave-function is a plane wave with wavelength ), enters a regionof constant potential V>0 where the wavelength of the particle is 24, . The ratio (V/E) is: B (GATE 2003] @iR oe ais Common data for (GATE 2004] ticle is maximum at ound 4 4 rporannanpesoce particle inte mlon Oto Fi is: eee i unstouna SPRER-ENBERVUA where u(x) is areal funcisi This Wit EYORES Te SUH AREY” @ 2()v \ww.careerendeavour. cama, ) A one-dimensional harmonic oscillator carrying a charge-g is placed in a uniformelectric field E along funcion y (x) =u(x)exp(ix), [GATE 2004] positive axis. The corresponding Hamiltonian operator is [GATE 2005] wd? 2 wd? 1,4 +e +98 soot she? ~ gx Rpt + ab © omar gh 4 wed? 2 rail Oni at? dia? + gbx Fy Srt gh ae A particle is incident with a constant energy E ona one-dimensional potential barrier as shown inthe figure. [GATE 2005] ‘The wavefunctions in regions | and Il are respectively farderavine oerillatare c re] 4 tial s(the jionof is: igure One Dimensional Potential qa 10. 1. 12, 13, 4, (©) oscillatory, oseillatroy c (©)oscillatory, decaying @ decaying, decaying, Aone-dimensionatharmonic oscillators inthe state v(x Falivel®) ~2y (x)+¥2(x)] Where yo (x), (x) and y,(x} ae first excited and second excited states, respectively. ‘The probability of finding the oscillator in the ground state is: {GATE 2006 3 9 @o OT On @1 A fiee particle s moving in-+x direction with a linear momentum p. ‘The wavefiunction ofthe particlenormalised inalength Lis: [GATE 2006] ~2L to - Where Y isa constant : : [GATE 2007] The normalization fi @ £ ‘The energy eigenval ve ots Mesto CBPERTEMMRESOGH COM fh @o wm on Om ‘There are only three bound states for a particle of mass m in a one-dimensional potential well of the form. shown in the figure, The depth V, ofthe potential satisfies [GATE 2007] Lay, anh? ee nh? anh anh? nih? we SOx”? @) Ar L, there is tue Vill OSS OBOMTORTE OIF LA)IA] and A, denote the ampitudes for the incident, reflected and the transmitted waves, respectively. A The ratio of the reflected to the incident amplitude “* ° A a) 1s (b) VI=T in magnitude | i | Aq) )_E | wath [asl |_ i (©) Areal negative number of [al ke | ‘Throughout 0 L. (©) For, x<0, the probability current is S,(I+R) (@) For xL, the probability current is complex. 21. Aparticle is placed ina one dimensional box of size L along the x-axis (00 as shownin the figure (GATE 2011] One Dimensional Potential ° v 3 Vix) Ve +E 1+ 3 For E 0 is ofthe form (@) (b) ee (©) ein Oem ‘where cis areal positive quantity. 26. Apatticle is constrained to move ina truncated harmonic potential Well (x> 0) as shown in the figure. Which one of the following statements is CORRECT ? [GATE 2012} wl 3 (a) The parity of the fits tate is even 1 (©) The ground state ee} nergy is ho 27. Acone dimensional ha Gate, [n), given by ‘The average energy of r 5 t [GATE2014) | | 28. Apartcle , is confined inal i x= 41, Another Particle P, is confined in a one-di mparing the two particles, one can conclude that : [TIER 2010) i (@) the no, cr SEE cn R. } () the no. of nodes in then x A ri ie (©) the energy of the n'" level of Ps the SanBaS THAPORP I= (the energy ofthe open? GareerERceaPOUr.COM I 29. ‘The wave function ¥ of a quantum mechanical system described by a Hamiltonian { can be written as a | linear combination of ©, and , which are the eigenfunctions of H with eigenvalues E, and E, respectively | Att=0, the system is prepared is the state ‘Y= fo, + 20, and then allowed to evolve with time. The | i th (B,-E,) ‘wavefunction at time T= will be (accurate to within a phase). (THER 2010] 46,3 at ee @ sitZO (0) o, © sa-5% @o, 3 4 2 © zMFZM — HZ One Dimensional Potential & rtial 30. Two identical non-interacting particles, each of mass m and spin 4/2, are placed in a one-dimensional box of Jength L. In quantum mechanics, the lowest possible value of the total energy of these two particles is ¢y If, instead, four such particles are introduced into a similar one-dimensional box of length 2L, then the lowest possible value of their total energy will be (TIER 2011] (@) 26, (b) Sey (©) 3ey2 @s, 31. A particle of mass m is placed in the ground state ofa one-dimensional harmonic oscillator potential of the form. V(x)= 4? 2 where the stifiness constant k can be varied externally. The ground state wavefunction has the form “hich w(x)ecexp(—ax? Vic) where a is a constant. If, suddenly, the parameter k is changed to 4k, the probability that the patticle will remain inthe ground state of the new potential is (TIFR 2011] (a) 0.47 (b) 0.06 (©) 0.53 (@) 0.67 (©) 0.33 0.94 A _qpuceccamscctiarit -Ix ; 32. A particle in a oneélifnensional potential a ) where a is a constant, It foot ITIER 2012] (@) Vox? 33. A harmonic oscilla i : P Vee . 1} where @, (34) is the Me [+>]. The expectation i ' J) eP, is 8, oe value (£) of ene (TIER 2013] @ 1.58Ko — 146 ho 34, A particle is contin f gsi dtl 184 Vishingly thin but strongly repulsive : partition is introduced nthe.exact:centre of the box, and the-particleés’llowed to come to its ground state, then the probability jc i [TLFR 2014] come necoiiy enRpCateerenieaveurcom | ! masa > stively ' ' | woot y ive? ‘ _ @ © ' | t Wweor i woot © t @ \ 1 One Dimensional Potential 36. 37, 38. 39. 40. ‘A particle is moving in one dimension under a potential M(x) such that, for large positive values of x, V(x) = ke, where k > Oand f > 1. Ifthe wavefunction in this region has the form y(x) ~ exp(—x*), which of the following is true? [TIER 2615] @a=Se1 () A=p (©) a=2p-2 wae ‘A one-dimensional box contains a particle whose ground state energy is €. Itis observed that a small distur- bance causes the particle to emit a photon of energy hv = 8 , after which itis stable. Just before emission, a possible state of the particle in terms of the energy eigenstates {y;,Y/z.-} would be [TIFR 2015] ya +, —Ayry + 5, V2y,-3y, + 5s ) a : (©) Val @ 6 1000 neutral spinless particles are confined in a one-dimensional box of length 100 nm. At a given instant of tie, i100 ofthese partici have energy 4, andthe remaining 900 have energy 225, then the number of particles in the left halfof the box willbe approx [TIFR 2015] @e2s jy,00 ‘A one-dimensional qu: [TIER 2015] requency «of the oscillator is denote the probability in each ccase that the system is uency change, which of the followingistrue? 2p, @ py The ground state (ap: ina one-dimensional potential V (x) is exp(-x? /2)}eoshy f2 aya), yrs 94 that ht =1,is (upto an additive constant). {JEST 2012] x x oF www.careerendeatourldain (© 7) Vietan (Bx) (@) 5 Vxcoth(V2x) ‘The grond state energy of 5 identical spin-1/2 particles which are subject to a one-dimensional simple har- ‘monic oscillator potential of frequency @ is [JEST 2012] (15/2) (13/2) , (1/2) @) () To ©) Ty (d) Sho tial One Dimensional Potential 8 0f] 41, A quantum mechanical particle in a harminic oscilato potential has the initial wavefunction w, (x) +v, (x), A *) where y, and y, arethe real wavefunctions in the ground and first excited states of the harmonic oscillator Hamiltonian. For convenience we take 1 for the oscillator. What is the probability density of finding the particle at x at time t= x? (JEST 2013] ©@) (vi@)-vo0)? () (vi)? wold} stur. ona © (V+ v0) © (viCoy' +¥«(@)) 42. The lowest quantum mechanical energy ofa particle confined in a one-dimensional box of size L is 2 eV. The energy of the quantum mechanical grourid state fora system of three non-interacting spin-1/2 particles is (JEST 2014] (a) 6eV (b) 10eV () ev (@) eV ae 43. Consider a square well of depth ~V, and width a with V,a fixed, Let ¥, > co and q—» 0. This poten- tial well has [JEST 2014] (a) No bound states _.(b)sI-bound state... (©) 2.bound states n.(d) Infinitely many bound states eT tor is each ofthe | \ ential ditive | shar- www.careerendeavour.com 4.1. Orbital angular momentum : Tn classical mechanics, the angular momentum Z of particle is defined as Replacing the position’ can have i 2) E 0 59 70" sind55) i fo 'b6 www.careerendeavour ‘Commmutation Relations [2..£,]=iné, [é..£.]=int, [é,£.]=-ini, ind, [é.£.]=-iné, ic. the components ofthe orbital angular momentums cannot be measured simultaneously accurately. [Z.é]=0 [2.4,]=0 [2.i]-0 i.e. square of the orbital angular momentum commutes with any one of components of the orbital angul momentum. [ina]e0 [i-A.J-0 Angular Momentum & Spin [E., Eigevalues and eigenfunctions of £? and £,: can be measured simultaneously accurately | Since, the commutator bracket [ 2°, £, ]= 0, therefore 2 and and both have simultaneous eigenfimctions or eigenkets. Denoting the simultaneous ejgenkets by !2,m,) (where Vand m, are the orbital quantum musaber and orbital magnetic quantum number respectively), the eigenvalue equation for ? and £, can be written as P\l,m,)=1(1+1)4 |l,m,) 1s, Wel where Yon(914) | where ¢ =(-1)" Raising and Lower | Raising Operator: wirwoabeerendeavour c corn ) Lowering Operator: i Important relations: [énd, J=nd Actionof Z, and Z_: neu E,|um) = (@=m (+m, Dm, +1) i = (Comm) Yo (8,9) ie. yr Angular Momentum & Spin | Ar E l,m) = (04m) (T= my, #1) |L,m, 1) sol ie. LY,,,(0,6)= (e+ m,)(1-m, +1) AY, (8,0) Matrix Representation of the operators: Elements of 2 = (6 my sm) = e(e-+1) 1 See’ Sum, willbe non-zero for 1 =1' and m, 2 wo o 100 B=|0 wo |=2n7]0 1 0] (orr=1) o 0 awe] [9 on Blements of £, = (€'mi'|Ee|m) =f See Sys, willbe non-zero for != "and m, = my, ie i 41, willbenon-zero for Sol 5 willbe non-zero for careerendeavour.com | v2A}1 0 0| (for/=1) 010 Expectation values: Sol Example 1: Which of the following are eigenfunctions of L,? For the cases where the fimction is an eigenfiuntion OfL,, find the correspondence eigenvalue. (a) sind e* (b) el) (©) eB sing (d) "cos Spin nition soln. Soln, é ag @ ning (sino ef) =(-in)sino (je =nsing We know, L, So, eee isaneigenfunction of L, and the corresponding eigenvalue is a “ng el) <(-iny(i)e@r4) = nel) So, el! wo isan eigenfunction of L, and the corresponding eigenvalue is ft. © ~nle® sing)=~ine™ cos So, e® sind isnot eigenfunction of L,, ay ME" 94 So, r* cos@ is an Example 2. (a) Is valies? sanaboms (a) We know that, i ‘Thus cos +sin@ é (osovsinoe wie a ans “] Thus, cos +sin 6 e'* is an cigenfisnction of L? and the corresponding eigenvalue is 27. LZ, (cos +sing é) = GL¥0 (8.0) +e2L,¥] (8, #) = 01.04 cghY (8, 9 sind ef ‘Thus, cos@ +sind # is notan “eet Lely You can write Ypo(@, 9) as |2,0). Also, L,|£m)=A,l(¢—m)(€+m-+1)|£m-+1) Soin. Soln. q@ Angular Momentum & Spin Ts, £,|2,0)=AV6 [2,1) a= etl 0) =) (8.6)= net (+ 1ca-S ho (6.8) = foe [St 3c0s? 6 ~ t+ ieot 0 (ost 8- i)|- ~ [sina cose Example 4, The operator associated with the measurement of the product £6 is (Lely +LyL,)/2, Calculate the mean value of £4 inthe state |é m) . Wehave L,=L,+ily and L.= Example 5. a mecspran Gerona 1) +}1,0)+|1,-1) J is (a) 2, 6 Be © 48, @o 4.) = 3[ (tales) +(1af2,ft) + (142,11) + (to, 12) + (tof, 1) + (oz, [1~1) +(1-1z,[tu) + (1=12,]10)+(1-2)2,)1-1)} L,, willraise the value of m to m + 1. Only those terms will survive in which this raised valued of min the ket is equalto the value of ‘nr in the bra. Thus, (ta) (eb9}olt—1)] $f (uA) «ofa = Ar ipin y2 eket Angular Momentum & Spin 4.2. Spin angular momentum : Stern Gerlach Experiment ‘The existance of spins first confirmed experimentally by stern and Gerlack in 1922. Using silver (Ag) atoms, Silver has 47 electrons, out of which 46 electrons form a spherically symmetric charge distribution and 47* electron occupies a ‘5s’ orbital (/=0). Ifthe silver atom is its ground state, then its total orbital angular ‘momentum willbe zero. thes Spin uy Beam — Spin down N Non-wniform magnetic field Ifabeam of silver atoms passes through an inhomogeneous (non-uniform) magnetic field (along z-direction), wewould expect the followis @ Classically, there yf sacra {ibe symmetric aout theundefeted direction 2 (@ Accordinh to S¢ ar momeritum then the beam will split into dise und state (/=0),therewill be only spot on the scree: 5 But, in experimentsit To solve this problem, to its orbital angular momentum , the electron possesse: ftum has no connection with the spinning motionof the electron fuected with an intrinsic degree of fal analog. Unlike orbital angular spaial degrees of eg about its own axis. 1 freedomie. spin. Consider particle AREER. CHDEARTTRS 1’, The magnetic dipole moment www.éaréefendeavour.com where [is the orbital angular momentum of the particle and z-components of magnetic moment will be For an electron the relation will be 42 =~ Ifthe electron is the eigenstates of £, ,then e =| Ina (4), (en pm where (2) = Wg = Bohr magneton and mis the orbital magnetic quantum number. Me Si Angular Momentum & Spin ah lt he. Fora particular value of J, we will get (2£+1) values of'm, ie m, veg Lyd Similarly, ifwe take electron as spinning charged sphere, then the magnetic dipole moment due to the spin motion ofthe particle will be where ¥ isthe spin angular momentum oftheelectron and g,, is the spin g-factor. Therefore, z-components of magnetic dipole moment willbe Ifthe electron isin an eigenstate of S, , then 2 fnere || — yp = wae ue te) (us), =-S.Ham, Fora particular value § Every fundamental Fermions: Parti! Bosons: Particles ha’ , protons, neutrons etc. r ete, : Photons, pions, ‘Therefore, there are “ @ Orbitalengular and itis characterised by two quantum numbers ies ‘Thus, the total angula. ues from -s to +s, and itis characterised Ree ican z-axis ie. ‘m,’ which can take values from -/10+7. In the Stern-Gerlach experiment, the silver atom has orbitel angular momentum to be zero ie /=0 but the spin angular momentum to be 1/2 ie. s=1/2. Therefore, total angular momentum will be 1a 2'2 =|!~s| tol! +s| For thiis, we are getting two spots on the screen, Pauli’s Theory of Spin : + Spin angular momentum has components SS, and S, and the corresponding operators satisfy the foll owing commutation relation: [8,,5, J=ins, [8.5] =in3, | A Bpin vin tum rtwo take atthe re fol xs volar Momentum & Spin | (8, 8.) i.e. they have simultaneous eigenstate as following: §?|sm,)=s(s-+1)A?|s,m,) 8, |s,m,) = m,h|s,m,) + Raising and lowering operators ate defined as following: §, +i8, 5, |s,m,) =hy(s—m,)(s+m, +1) |s,m, +1) S.|s,m,) =Ay(s+m, )(s—m,+1) [s, m,—1) 8, 1 a 1 + For a spin > particles ike electrons, m, can take values 3+ and ~>, so the possible spin states are Angular Momentum & Spin + Pauli Spin Matrices: Spin-angular momentum can be related with Pauli spin vector as follows: ate 2 O11 0 -i (i 0) a where Ox =| ) y= o2= are components of the Pauli spin vectors and known 10 io {o.-1) 4s Pauli spin matrices. These matrices satisfies the following conditions: > \ Si Angular Momentum & Spin town Soln. Soin, () o2=02 =o? =1 (i) 0/04 +040; [oj,0,}=215 Gi [0.9% |= 226 x07 (iv) Pauli spin matrices are hermitian, traccless and determinant is 1 (¥) Ox0y0, =i (vi) Since spin does not depend on spatial degrees of freedom then the components of spin 5, Sy» Sz commute with all spatial operators ie, momentum, position, orbital angular momentum, (vi) °° = eos tio; sina A\(68 Example 6, The wave fintion ofan electron at an instants given wy =f (r,0)e" x19 .Caloulate the (vii) For any two vectors 4 and Bi ( =(4B)1 +i0(AxB) ‘The average vend Now, cA The given wave fue is <2, This shows that it is an eigenfunction of L, shows that itis an eigenfunction of S, with eigenval 10 Lesme rere Therefore, He \orsiaareerpdeavour com So, (vluely) =e 2 lv) +(v lw) ]=3e5 Example 7, The spin part of the wave function ofa spin-¥4 particle is cos yyy. + sina e!9%-12 , The x- h component of the spin is measured, Find the probability of getting the result >. Here a and fare real constants, ‘The wave function before the measurement s| 71) =cosary,. +8inace” 7p h, ‘The wave function after the measurement will be eigenfunction of S, corresponding to the eigenvalue = ive. (42+ t12) Soin. Soln, <= Angular Momentum & Spin A So, probability of getting > inameasurement ofS, i | | ot 1 iB (alz)= Jp leosa rant sine x | 42+ #12) 1 ina et = pleaser aia |va)+sinae? (x yal z-u2)] I i = zz (cosatsina e ) ‘Thus the probability is Kalae 4 (cosc+sina e?l(cosa+sina e®) =1f142c08a sine cos 6 2 We can get the eigenvalues are aumple 9. Clete waltiterndeavorns com (LSv)|¥ (0,6) x02 ]= bal? (0,0) Setira 130 (0,0) =( AE) 9 (6,6) Hf La G9} 1.10 (0.4)] =5[ ian 4) '(6,4)]-E [nt @e)+1'@.9)] and Sutin eA en Theirs, (45,)[1" (0,8), Je ealW (IH, Jere Sol Example 10. A particle isin an eigenstate of J, Prove that (J,) =(J,)=0. Also find the value of (J?) and (J}). Soln. Let the eigenstate of J, be | jm) Spin | Angular Momentum & Spin (4.)=(imi(I45.)Lim)=2 mJ, m) +2 (jm J. jm) = I FD= mw hj jam ot) +2 (FFD) — mbm hj | j,m—1) = since (jm| j,m+1) = (jm| j,m-1)= Similarly, we can get (J,)=0 ‘We know that, J? +J7 Therefore, : i 5 It is expected that ' : sesare i Example 1. The Show that (A) Soln. (6.4)(0.B)=(6, A ith those of o. . =074,B,+054,B) +0,0,4,B, vod i Using the relations o dni Caredtendeavourcamn © 6,6, +0,0, =6,6,+0,6, =6,6, 4,0, =0 ‘We get (6.4)(.B)=(4.B)+i0,(4,B, ~ 4,B,)+i0,(A,B, - 4,B,)+i0,(4,B, ~ 4,B,) = (AB) +io.(AxB) Example 12, Find the equivalence of the following operators: @ S2S,S2; (di) S7S7S?; (iii) S,S,S? (J ose elo) 6 = Soln. (i) $25,5? =| io otte,(* 3 ‘ 2) Example 13, Consider a spin (1/2) particle of mass m with charge -e in an extemal magnetic field B. () What is the Hamiltonian of the system? i) IES is the spin angular momentum vector, show that (asa OP) Therefore, = de 5.0.) (558,]+(5.52] Fg M[(S,2. 8.8, )?+(B,S,~B8,) 34 (5.8,-8,5,)F] e £(548) Example 14. Consider a system of two spin-half particles, in a state with total spin quantum wamber S = 0. Find the eigenvalue of the spin Hamiltonian H = AS,.S,, where Ais a positive constant in this state Soln. The total spin angular momentum S of the two-spin system is given by Angular Momentum & Spin L Eigenvalue of 45,5; = a? (+) a4) oce/ore ~4 ‘Example 15, An operator P describing the interaction of two spin-hal€ particles is P= a+b¢j.c,, where a, b are constants, with ¢, and o, being the Pauli matrices of the two spins. The total spin angular momentum $= 5, +5, = (1/2)A(6, +6,). Show that P, S* and S, can be measured simultaneously. Soln. P, S? and S, can be measured simultaneously if (P, 7 ]=(P,5,]=[87.5,]=0 ‘We know that [S?, S,]=0. From the definition and [S,,P]=[5, Since $* and S, cox mw neously, Example 16. ovaitelelf Halihd magnetic moment 11 inan extemal eg Utcsenerg pure” Soln, The magnetic moment of the electron is given by ‘The Hartitorian iB = —1,B, = 150.8, For total applied magnetic field B = B,)-+ B,2, the total Hamiltonian H=u,(0.8, +6,B,) ‘From equation dui 1 Hat =o B, Ge eet Glos (6.8, + 0,8,)] Soln, Soln. gq Angular Momentum & Spin 18,3 +10,,0,]8,2+[6,.0,]B,2 = Zullo B,-0,8,)8-0,8,5-+0,8,21 i AG oe e S x B]=-=3[Bxo] =<[Bx = hal [ox B]= at [Bxo] ae HI] Which isthe time rate of change ofthe magnetic moment, Example 17. The slate Cen EBABY levels of the spin-orbit interaction Hamiltonian 47.5, The 2p state means s| Therefore, the corresth Using equation (ji)-(v), FaSener 3 GARE Hae Www. care! serendeavour, com 43 We ~ 2h -ye =-Ane D Example 18, The Hamiltorian of a system of 3 non-identical spin-half particles is H=AS,S,— BUS, +5,) 8, where A and B are constants are S,, S, and 5, are the spin angular momentum operators. Find their energy levels and their dégeneracies. Writing S=5,+5,+5, and S,, For For +S, we have ieee S? = $2.45? +28,8, = SS = 3 (8 Sa -S3) Similarly, S,.5, (Siz - 5 -S2) y AL Sol Spin tction nergy Angular Momentum & Spin Soln, since s,=1/2 and s, =1/2, the possible values of the quantum number s,, = 0 and 1. When s,, = 0, the possible values of $= 1/2 and 1/3. The Hamiltonian becomes H= 4S,S,-B(S,+S,)5, 4053-89-84 BS" ~83-8}) The energy is A itsabsn #1) 95,41) =9265, 4D) + 2M E(641)~ 54650 += 565,40) Since s, = 5,=5,=1/2, ” 7 +) + be [ac D~552+1)- 3 = | i fH where A and B are ms “ENE EY Tal of the two spins. Find the energy levels of the system. Total spin angular “at rere recom ae 5,8, -1(s?-s?-8? x 2) Since 5, = 1/2 and s, = 1/2, the possible values of s are 0 and 1. The Hamiltonian, 17 = A(S,, +8,,)+B(S,S,) ~ 48, +2(S?-S}-S2) The corresponding energy will be Bap 3.3 Ey, = Amp oH [ss+-2 3| 3 When s = 0, the possible values of m, = 0, then Bo =~ BR, By When s = 1, the passible values of m, = 1, 0,—1, then By, = 4h+ 7 ie Angular Momentum & Spin Example 20, Fora spin 1/2 particle, the expectation value of S,5,8, is 7 in ¥ : oF o-> om O-% sm sssctoreTlt ol ole all 1] (855)=(eB55ly) “Fier, LPs # 00%, ay Soin. rampienn ance CASE a DER! vg Uy is in S, sepresentation. What isthe probaitty MAMAS GALGRDRRO RAMONE ellen? 6-)8]26 | °] el Soln. atte? aq 0 +e, il 1p AD 4 1 sng longt 1 Probability offindingz-components ofits spin in~zdinection = isi Example 23. Suppose a spin ¥4 particle isin the state |) = K A h What are probabilities of getting + and ~ zit (iz component of spin is measured? " So Spin Angular Momentum & Spin (i) xcomponent of spin is measured? (ii) calculate the expectation valueaf S, ie. (S,) Soln, (Given: |g) ee Hach 2a Probability ofgetting S, = the operator eft si i an. careerendeay ous ory) B [CSIR June 2011) wt Tons (@ Teos+ i6.B ©) Isinot+eosaat @ Tsin2at +" eos 20t H, Soln, Given operator: _ Using the identity, e* = 1cos0 +i fisin® (where f is unit vector) , we get Ay ea a Bei eB = 1008 wt +i6.B sin of = I cosa +i sinat Correct option is (b) Soln. Angular Momentum & Spin 1 al Ina system consisting oftwo spin = particles labeled 1 and2, let Sal) and §(2 7 2) denote the corresponding spin operators. Here 6 =(6,,0,,0,) and 6,,0y,¢, are the three Pauli matrices. [CSIR June 2011) ‘A. Inthe standard basis the matrices for the operators St” 81) and s{s(”) respectively, La 0) #(-1 0 wi 0) #(-i 0 @Flo -)4lo 1 OF 0 a) alo i o004 000 -i 01 (0 i 0 oF #10 0 i 0) lo O - Oo} wil o #/i 0 00 OF ‘ . @ 7 a 4/0 100 4/0 08 4/0 00 -i/ 4/0 0 018 i 0 0 0} ioo o ooigd oo10 B. These two operators satis g(2) (2 @ {990,99} € (No(2)_ githe(2) = o[sts?3090)] A. PauiliSpin matrices: BRE Correct answer is (c) B. [sss] =s1050(50 -—ss)"st50? fo 0 0 -i) (90 0 -i) (00 0 -i) fo 0 -i) #/0 0 i O)0 0 4 0) MIO O ~i Ojmlo io “4/0 -i 0 o/4fo i 0 0} 4/0 i 0 of4jo i 0 0 i ooo) ooo) tioo o) li oo Correct answeris (a) An Sol Sol Angular Momentum & Spin 3, Ina basis in which the z-component 8, of the spin is diagonal, an electron is in spin state _{a+i/V6) vl a ] The probabilities that a measurement of S, will yield the values 1/2 and —h/2 are, respectively [CSIR June 2013] (@ 1/2 and 1/2 (b) 2/3 and 1/3 (©) U4 and 3/4 (@) 13 and 2/3 Soin, The eigenfunctions of S, in a basis in which S, is diagonal are 1) a la)=( 0) for 5 eigenvalue and |4,)= aig () for-2 eigenvaue ‘The given state can be written as linear superposition of |g,) and|¢,) as (1-4i)/16 )oceceamnernmcmrenennse 2 3 The probability of idl ' i 3 i q i and the probity 4 Correct answer Is | 1 seule AREF he Aspin —> partic CAREER es S? and S,. If we measure S, the Vi probabilities of getting wun careeren¢ cay our.com ICSIR Dec 2013] lod 2 3 L3 @) 5 and > OMT (6) 0 and 1 ORmMa Soln, 1 hei Given : Asin 3 5 particle is in the state y = Vil 3 fi & Angular Momentum & Spin 9 iv 2 Probability of finding S, as 4 and ~~ are (Gj? =f and |c2)? ==. respectively. ‘Method-2: 1 (+i Initial state of particle = |i) “Fils 1 0 Final state of particle = “en and |, [I] Probability of finding S, as +4 and ~> are |(ve, wif =yyand (ve lvyf 3 respectively. Correct answer is (b) 5. Let iu, denote the eigenfunctions of Hamiltonian for a spherically syrnmetric potential 1). The expectation value of L, in the state [CSIR Dec 2013] Soln. v $[v. set ae ee. in usual notation), is [CSIR Dee 2013] @ wt -3) wo @ tins oma Soln. Given: vate. 1+ V5 v210~ Vitva,41) (te)=(V|bely) vail lvanal-vT aul ee +2.)] Sol Angular Momentum & Spin :013) | Soln. Now, we know, Ly nem = (E=m)(E4 +1) AY peat LM ptm = E+m)\(E=m + RY ptm ae a(as sie, (2)=(2 )ypeuddevenseradestibur.com For a given /, m varies from ~! to J (Bg, overs when mao => (22), =2e(e+1)i? 2 2 2\ ade (Ez) yy cecurs when mae => (2) | aban Correet answer Is (d) Consider the normalized wavefunction § = a,y,+a,Win +aj¥.. Where V,, ia simultaneous normalized eigenfunction ofthe angular momentum operators Z? and L., with eigenvalues 10+-1)8? and mf respectively If 6 is an eigenfunction of the operator L, with eigenvalue A, then ICSIR Dec 20141 = Angular Momentum & Spin Soln, Soln. 7 call = @4 v2 Oe © 4 a, a : i Given the normalized wavefunction: @= 4, yy) +4, Yio +45 Vier Now, £,g -|4 a Using the operations ofthe ladder operators ie. Levin = {l= m\(Le m+ )RY int Lovin = J+ m)= m1) Vint = a Vita, Vora Vir] Correct answeris Oy rs Let 65(6,,0;,0,), vectors in three dimensorss AREER EAD ft ee “= careerended¥re3m Seal © @b)I (@ layer Letassume, G=ai+a,jtak and babi +d,j+by6 ‘Therefore, [8.4,6.6]=[20, tac, +qo,ho, +ho, +0, ] “[oe.he,)4[2.0h0,]+[4eh0.]+[a0,,bo, Jeo Using the commutation relations, [0,0 ]=2io,,[0,,0,}= 210, [0,0] = 2iy we get [4.6.5.6] =2i(ab,0, -a\bj0, 4,9, + a,b, + ab, ~ 30:0) = 24 {(a,b, ~ashz) 0, ~(aybs -ash,)0, + (aba — ab) 0} Spin | Angular Momentum & Spin astant | | 6. satrix) PRACTICE SET Let L=(L,,L,,L,) denote the orbital angular momentum operators of @ particle and let L,=L, +iL, and L_ =L, ~iL,. The particle is in an eigenstate of Land L, eigenvalues ¢(é+1)4? and Ag respectively. The expectation value of L., L_ inthis state is: (GATE 2001] @en 208 @o @en ‘ 12. Anelectron is in a state spin wave function 6, = (4 } in the S, representation, What is the probability of finding the 2-componenet ofits spinalongthe -2 direction? =~ [GATE 2002] @075 (6) 0.50 (00.38 (0.25 Ina Stem-Gerlach experiment, the magnetic field is in +2 direction. A particle comes out ofthis experiment in |+27) state. which: ofthe following statements is rue? [GATE 2002] (2) The particle has : (b) The particle has entum. (c) The particle as turd. (@ The particle has $e x6 fF momentum. [GATE 2003] (2) 0,0,07/4,n7/4 FE 9-/4,97/4,0,0 The commutator, [9 Yin (0.¢)is spher 2 i (@) e(241)h f ~ ad) +£h Fora spin 1/2 partie, 5, arespin operators, is e , [GATE 2005] in prea eames {meee # oa wiweareererfdavour.con? ~i The commutator [L,,y], where L, is the x-component of the angular momentum operator and y is the y- component ofthe position operator, is equal to: (GATE 2006] @o (®) inx (6) tty @ itz ‘Which one of the following relations is true for Pauli matrices? {GATE 2006] @ 5,9, =9,0, (0) 9,9, (0) 9,9, =io, @) 9,6, =i0, Let |v) denote the ground state ofthe hydrogen atom, Choose the correct statement from those given below: [GATE 2008] @ [Luly ]Ivo)=9 © Fy)=0 © [LS]ivo*0 @[S,.8,]}y.)=0 Angular Momentum & Spin 10. ul 12, 13. 45. 16, 17, 18. 6; (i= 1,2,3) represents the Pauli spin matrices. Which one of the following is NOT true? (8) 5,5, +60, = 28, (©) Theeigenvalues of 6, are +1 For a spins particle, in the eigen basis of $7,5, the expectation value (sm Ist |sm) is [GATE 2010] (b) [s(s+1)-2m* |x? () [s(s-+1)~ ~ IfL,yL, and L, are the x, yandz components of angular momentum operator L, thecommutator [ L,Ly, L, Which one of the following commutation relations is not comect?carmicwr A spin-half particle is ip and spin-down states. If tty and i) are the cigen to one decimal place, of the operator 100, +50, If Z is the orbital an, then Z.§ does not commute Wis! El ndcovoure com 0 } and () represents the spin-up and spin-down states, respeetively. When the system isin the state y, its probability to be in the spin-up state is If L, and L_ are the angular momentum ladder operators, then the expectation value of (L,L_ + L.L,) in the state |/ = 1,m= Let Zand f be the angular and linear momentum operators respectively, for a particle. The commutator A 19 20 ai 23 4 Spin | Angular Momentum & Spin 19. I 20. 010) rh} | on, I 1 2. Iti) of the q mute U] i a | 23, sin the | LL) q autator a ‘The Pauli spin matrices for three spin-1/2 particles are 6, and G, respectively. The dimension of the Hilbert space required to define a operator O = 6,.(6 x3) is [GATE 2015] ‘An operator for a spin-1/2 particle is given by 4 = A6.B, where B= ali 5), & denotes Pauli spin matrices and 4 is a constant. The eigenvalue of 4 are IGATE 2015] AB @*y () 2aB (©) 0,4B @) 0,-aB Consider the Hamiltonian H= fez Here 3 is the position vector, fis a constant and G =(0,,0,,0,), where o,, 0,50, are the three Pauli matrices, The energy gigenvalues.are,, aeRO [TIFR 2014] ee “1 @ f(\verrs iy) © #6 @ af (x+y+2) Ina Stemn-Gerlach| nt waithSpi : i |to form two spots on the screen, one directly above er. TI ef iio fe fereen at the position of the upper spot. The particles {hatgo ti ole are then passed t! ther Stemn-Gerlach apparatus but with its magnets ro 90 unt of the beam direction, Whichof the following sho appei e. (TIFR 2015] 4 -e+— e-| @ | gy www.careerendeavour.com An rigid rotator has the wave function [TIFR 2015] V0.9) = N[21¥, 0(,9) + (2+ 0%, (6,9) +32¥,(8,0)] where ¥, (0, ) ate the sphereical harmonics, and Vis a normalization constant. If Z, is the orbital angular momentum operator, and L, = L, 4 iL, the expectation value of L,L_ is 210? 23h" 2sn* oA om OA @o Define o,=(f'+f)and c,=-i(f*~f), where the o' are Pauli spin matrices and f,f* obey anticommutation relations {f,,f}=0,{f,f1 =1}. Then o, isgivenby [JEST 2012] (@) fi fnt 2st f-1 © aft fat Oss 25. 26. 21, 28, 29. 30. Angular Momentum & Spin Consider a system of two spin-U/2 particles with total spin 3,, where s, and s, are in terms of Pauli matrices o, . The spin triplet projection operator is [JEST 2012} dass ») 3-5. S45. _ @) gts 0) 47-55% © F7t5e O75 a of homogeneous magnetic fed of magnitude B along z-axis which is prepared initiallyin a state [Y) = allt) |+)) at tinet=0. At what time‘? wil the particles bein the state |v) (ay is Bohr magneton)? (JEST 2012] to Zh 4) 022 fae a @'" TB TB © 2u,B (@) Never IfJ,J,.J, are angular momentum operators, the eigenvalues of the operator (J, +J, )/ Aare % ae, [JEST 2013] (2) real and discrete wit ional spacing the L basis of states with m= ‘after the measurement? {JEST 2013} 1 fi/v2 @|0 a 0 v2 What are the fa waamercorenioayGurectaen and &,isa vector? [JEST 2013] @a,+a,andc, (b)a,+a,andia, (0) #(a,+0,+0,) (@) #ai Consider an eigenstate of Z? and L, operator denoted by |£,7m).Let 4 = ff denote an operator, where j isa unit vector parametrized in terms oftwo angles as (n,.",,1,) = (sin8.cosg,sin 8 sin 4,cos@) . The width 44 in |é,m) state is: [JEST 2014] @) LC peos0 () {Gre cos (©) yele+i}-m tsing (d) fe(2+1)=m* hos Spin "Pauli which vetate, Angular Momentum & Spin = 1 31. Suppose. spin particles in the state y=]! vi Ve 2 If S, (x component ofthe spin angular momentum operator) is measured what is the probability of getting to [JEST 2014] 1 2 = 1 On oF © oz CAREER ENDEAVOUR, puisasaceeponmmnmn es www.careerendeavour.com Three-dimensional potentials 5.1 Three- mensional rectangular box : We consider a free particle of mass m confined to move non-relativstically in a three-dimensional potential box ofsides a, band parallelto thex, yand z-axes respotiey Since the prices fie, the potential insiette always a positive pani js ecrnmemmzensrerst Fi Iv be the velocity, the pallid, “kes Sine the potential ener {apy ais eereideaycuarsemntcsmte time -independent 3- i D wave equation to obtain the wave functiuon y (x,y,z) and the eigenvalue ofthe energy of the particle. ‘The three-dimensional potential box has the following characteristics: 0, for 0a, y>b, z>0 a The wave equation for the Particle: ‘The wave equation describing the motion of the particle can be written as Q) albox de the aich is @ Three Dimensional Fotentiale & As V(x,y,z)=09 at the boundries and outside the box y(x, y,2}=0 at the boundaries and outside potential box. The boundary conditions can be writtenas, ee cacamecae| y=Oandy=b zeOandzee| Using the separation of variable technique, we caan assume the solution of equation (2) to be ¥(sy.2z)=X(s)¥(y)Z(2) 6 ‘Substituting equation’ 2 in equation (2), we get yet Say th, xY ae dy? Dividing throughout by XYZ, we get @ o (6a) (6b) (6c) Solutions of equations (6a), (6) and (6c) are X(x)= 4 sink,x+B,cosk,x where k, ¥(y}= 4, sink,» +B, cosk,y where k, = Z(z)= Asink,z +B, cosk,z where k, = Boundary Conditions: =0>B,=0 =0 => Asink,a=0 sink, (ii) Z(z=0)=0 => B,=0 Z(z=c)=0 > Asink,a~0 =>sink,a=0 k=" (y,=1,2,3 a ‘Therefore, the solution will become ne U(x y.2)= 4A, As sin sin ysin 2 @ c ne 2 (5392) = Asin xsin AZ ysin a Now, the eigenfunctio v(s, ‘Therefore, the total i CAREER ENGERVOUR) = w jes Ee) | 4} )-2] The integers »,,,, 2, are called quantum numbers which are required to describe the stationary state ofthe particle, Since for any of 7,,n,, 7, being zero, w(x,,z)=0, ie., the particle is absent in the box. But physically this isnot acceptable and hence any of 7, "y+, cannot be zero. For the ground state, n, =, =, = 1 and the ground state wave function of the particle is, Bae Vg RE in sin 2 sin = | abo abe Vin y2) @ ®) ‘Three Dimensional Potentials ofthe But and the energy associated with the ground state called the zero point energy s given by, 5.2 Three-dimensional cubical box: Ifthe potential box be cubical instead of rectagular, then a= = c the eigenfunction corresponding to the state of the particle will be represented by and the conesponi Ground state (in, ‘Wave function: ¥; Momentum : 71, www. careerendeavour. co ne my a, Wave function: Yau = Sin sin ™ sin 2 17 7s 12 vay = fein sin in 112 Bmx. ny, Wnz Vina = fy sin sin sin = BOs eo mh Momentum: Pass = P21 = Pia = V6 2_2 anne, 12 at Energy: Foi = Fi24 142 Soln, Three Dimensional Potentials Degenaracy Corresponding states (n,n,n,) 2 pré S11), (151), (115), (333) 26 =6 (433), (413), (341), (314), (143), (134) p=3 (422), (242), (224) =3 (332), (323), (233) =6 (421), 241), (124), 142), 214, (412) =3 (331), (133), (313) =3. GID, (4h, (14) =3 (322), 232), 223) Energy 7 gt | 1 P=6 (321), (132), (223), (312), (231), (123) in the units of = ( 2ma’ 1 p=i (222) " —— p=3 Gil), a3», 13) 9 p=3°— @21),(122),@12) Degeneracy: i The property that, twé different eigenfunctio Example 1.. aoe inthe ground ate and Ground state: n, = 7, ceca cn ail ererendenourcon th pth ph 24 Ry at = B= Bs. x10 ig - tay SENG = NB = SNF 5.1210 hg m/s Energy ofthe electron is ap? Egy FE a 181x10 7 =113 27 me First excited state: ee 2140 121 ie ah 24 Momentum ofthe electron: P21 = Piz ~ Pui =~ V6 = 8.1310 kg —m/s Sol Sol Sol 34) 12) 2) sand rethe ‘ctron eee ea) Soln. Soln, Soln. wen © 2ma? Example 2. An electron is confined in a cubical box of side 1A. (i) Find its lowest energy (ii) what is the ‘temperature at which the average ofthe molecule of a perfect gas would be equal to the lowest energy of the electron (k= 38x10 J/K)? (i) Ground state energy of the electron 6=362x10°Y J = 226 eV Energy oftheelectron: Fp... = ia, 3x? (1.05410) 249.1 e107 x(10)° (ii) I€T K be the required temeprature of the molecules, then =18.03x107 8 cle isin an eigenstate with energy @ rsa Example 4. Find the number afenergy states and energy levels inthe range # <(15h? /(8ma*)] ofa cubical box of side a For a particle in a cubic box of side a, the energy is given by Foo (ni +n 6m) == (m2 + +m) 2ma' Comparing with the given expression, we get 2 +n? +n? <15 The number of possible combinations of (1,1, ,) is (it 1 way 12,020,211) 3 ways (113),030),611 3 ways (122), 212), 221) 3 ways 22) 1 way eS ‘oe Deronaional Potala Soin. (123), (132), (2.13), 32 1), G12) 6 ways Total 17 ways Hence the No. of possible states = 17, The No. of energy levels = 6. Example 5. A crystal has some negative ion vacancies, each containing one electron. Treat these electrons ‘as moving freely inside a volume whose dimensions are of the order of lattice constant. Assuming the value of hitice constant, estimate the longest wavelength of electromagnetic radiation absorbed by these electrons, ‘The energy levels of an electron in a cubical box of side a is ie =F anton), ma 2ma’ ” Lattice constant a= 1 A = J0-1° m, The energy of the ground state is given by =1,2,3,.4 We considera particle On CAREER ENDEAS nF soap SES Ds SEU R along thes» zomg aie sip deutvergt: ae -okyyk, ar the force constants and ©,,@,,0, are the angular frequencies along x, y, axes respectively. Such a systems called a three- dimensional harmonic oscillator. ‘Again, and F,=- 2 where the potential energy Vof the oscillator i, 1 2 V(x,9,2)= 5 (kx? +h? +k27) ‘The time-independent three-dimensional Schrodinger equation for the system is w Fv (sy2) +5 (bes? +h? hz") v (5-2) = BV (02) \3 als tants wee- ‘Three Dimensional Potentials [2 (aix? +a3y? +032") ]y =0 @) me VF where, ape Equation (9) can be sotved by the method en ‘of variables. The energy eigenvalues and eigenfunctions ofthe three-dimensional oscillator are os E -(n-4 +5 5)po. E, -(1,+4}p0, where, Mgy m, and X(x)EN,, Therefore, the co aE: Forthe _ REEL AE com sse[nctyene3fion(oe3)pe where =n, +n, +n, is the total quantum number andn=0,1,2,3, It can be shown that except for the lowest energy level (n, =n, =, = 0,ie.n=0) allthe energy levels are degenerate, ‘The zero point (or, the ground level) energy B, for 11, = 3 Sho 2 ‘Thus the zero point energy ofa three-dimensional isotropic harmonic oscillator is three times thatfor the linear harmonic oscillator. the ground level of the oscillator is thus non-degenerate, =, = 0 ofthe oscillators given by S a The state with principal quantum number n= 1, there are three levels each of energy 5 %@ 2 Three Dimensional Potentials So, the first excited state is 3-fold degenerate. Similar, the state with n=2 is 6-fold degenerate. Thusall the energy levels of the isotropic three-dimensional oscillator are degenerate and the degree of degeneracy ig Flme(nea), Figure shows some of the energy levels ofthe oscillator. n, n, n,|n| Wave function] £, | Degeneracy(p) 0 0 O}0]~ veo Sho P Te0Rs0 Vioo o 1 ofl Vouo ry ae RY eS CS — t 1 1 She 6 i 3 % q o im Yoo. 7 DEAVOUR 7 STE ), (0, 0,3) (1) (1, 0,2) 2,0,1); 2, 0), 0, 1,2) States 10 7 n=2 (2, 0, 0), (, 2, 0), (0, 0, 2) Bo | States 6 Sho 2 00,0) States 1 Sho 2 G2, 1, 0), (1, 0, 1), 0, 1,1) 2, 0,0), (0, 1, 8}, (0, 0, 1) States 3 ia tials first level wey is ‘Three Dimensional Fotentiale iS 5.4 Spherically symmetric potentials : We consider a particle of mass m moving in three dimensions under the infiuence of. force exerted by another particle located at the origin. The force is represented by F =-VV (7), where, 7 is the distance of the particle fromthe origin. If V(r) depends only on rand not on the direction ofr. Such a potential, which depend only rand not on @ and @, iscalleda spherically symmetric potential. ‘The time-independent Schrodinger equation, fora particle moving ina spherical symmetric potential, n polar coordinstes is given by V?(r.0.8)+ Sev (rw (r.08 Now, y? oe = ay vat 9 where, R(r) and Using equation (13 We get, ‘www. wa career pau com D@¢)sin2@ Og? Se le-V(n)]=0 (14) Tnequation (14), the second and third terms are: oS ‘ofr whereas the first and fourth terms dependent onr, So, wevan write, 1 [z 2 (sno 24)) Q(64)| sind 20 20 as) 1 df par 2naGee . and Roa" 2) DM a6) where, Cis a constant. ‘Three Dimenelonal Potentials ‘Solution of the Angular part: Equation (15) is independent of V (r), so the solution giving the angular part ofthe wave function y (8, $) will be the same for all spherical potentials. Further, equation (15) is related to angular twomentumin the following way: O(. 4 2 1 @ Fine Sarg [1 af, ,62(d, #8 so, Balada te (one O82) rr - =-#00(8,4) Therefore, 1728, )= 2(8,4) (17) Clearly, equation (17) is the eigenvalue equation for the operator Z? -(2(8,¢) and 2, arethe eigenfunction and eigenvalue respectively belonging to the operator 72. Thus the normalised f wa Some of spherical har For£=0: (64 For For é#2: 13) og a3 x() smroe “a 4) sin cosbe"# (ar 12 cog? 5)? 300s? 6-1 #(0,)-(2) ae 1s? #ege(2) sin@cos@ e# 4a Ys? (6.9) as sin? @ e™ dale | three Dimensional Potentiale Nonmalization condition of the finetion ¥;" (8, 6): 9.9) 1 nthe J J Mn, (6.4)¥in, (8.6)sin@ 40 dp =1 a0 Solution of R(r) equation: The radial part of the Schrodinger equation is, 4 pa 2m ? E-V(r (41 “El GFE vlec=—ea 1d 2dr), {2m £(E+1)] =x9(r 2). {ee [E-V(r)]- = fa=0 (18) 1 Now, v(1.8,9) = R(r)©(8)©(4) = R(r) (0,4) =R(r) YP" (8, 6) ‘This is the complete three-dimensional wave fiction ofthe Schrodinger wave equation for a particle moving in ction a spherically symmetric M(). For a bound system, the wave function must satisfy the condition that, (r}>0,a5 r—>20 “ pcmemonromnants IV (r) isknown, thé, sxplicits a jince equation (18) depend onn and Jean have a discrete FeEo e2 r alu jscrete say, Ey, Ef, complete wave fundiigh E written as Here will be (2£+ 1) wave function , |) fold degenerate. The quantities number and azimuthal quanturn 5.5 Hydrogen ferent states, “The simplest of the i Se heel ih BUR eto moving around the nucleus containing only one ‘it nee era central Ais Sut of the Coulombian electrostatic attractive force. It Saamae -omplex atomic systems containing more than one elect CaP eta Bohr (in 1912) on the basis of hydrogen atom, Schrodinger gave the solution of the wave equation for hydrogen atom is 1926 formulating the wave mechanics. There was extensive development of quantum theory of the hydrogen atom by the subse- quent workers, viz, W. Heisenberg, max Born and Jordon and Pauli. The results of hydrogen atom was successfully extended to other hydrogenic atoms, The hydrogenic atoms are oftwo types: i Ionised atoms containing only one electron moving around the nucleus, which may consist of more than one proton and neutron. Singly ionised Helium (He'), doubly ionised Lithium (Li), triply ionised Beryllium (Be) are some of the hydrogenic atoms, ii) Atoms of, ions with closed shells and only one electron in the outermost shell form the second type of hhydrogenic atoms, Atoms of aksti metal and alkaline earth ions are the examples ‘The wave equation for the hydrogen atom: ‘The hydsogen or, hydrogenic atom is a two-particle system consisting of an electron of charge—e moving round the nucleus ofcharge +Ze, The electron inthis system moves under the central fore arising out ofthe Coulom- ic electsnstatic attrantion hahween tha nniclane and tha alanten tow al (st Three Dimensional Potentials ‘The potential energy of the system in absence ofany external field is, The potential energy is indepenent of @ and ¢ and thus spherically symmetric, Reducing the two body prob- Jem into one body problem, the 3-D Schrodinger equation in spherical polar coordinates can be written as 1 6/ 2)éy a 4 Sh) ow zal 6. + E-l V(r 0 Fan Var end 008° 56.) ant ap? * LEV = Marcelle +m, where # = is the reduced mass ofthe system . Using a technique, one can find the soluion ofthe above equation to be (18.9) =Ru (On (8), (8) W/careérendeavour, com Pr pita ya 30 aay 27- tert |e re B\ a) ay a5 ) ala) lee ia ‘tials ‘Three Dimensional Potentials = 2 rob] yf) nas Qo Rulr) 0.5 ° re o4 Ry (r) 03: ot \¥ é t 10.1 = ° fs I ‘Table: Real formo! 25 tron) for various values off, and mn | nt m {si TOG reese ar | www.careerendéavour.com os i 200 2s Ye = ag ger)” i oP gin o 21 1 2p Von aeqnee sino | BiG 2 eee eo P Vato War a? 21-1 2p von alegre me | Va00 = ane 1 (21- 18p +2p?)xeP? i 300 3s 2 when, p= 22, a, = & G&G Three Dimensional Potentials ‘Radial probability density : ‘The probability of finding the electron in volume element dr is given by Waal dt =|Vyiql° 17 sinOdrdede ‘Therefore, the probability of finding the electron ina spherical shell ofradius and thickness dris, au P(r)dr= J | Wael 7? sin@drdOdd =(R, fdr seogea Here, P(r) is called the radial probat ty density. 28 Figure : Variation of Rye (1) with rin units of@, Energy values forthe hydrogen atom: Energy eigenvalues of n state of hydrogen atom __ Hae 32 ehn® Sob ale | Taree Dimensional Potentials ss =~ 32 ern where, n= 1, 2, 3, ......... and is called the principal quantum number. ‘This shaws that the energy values for different quantum states discrete which is in agreement with old quantum. theory and experiment. Degeneracy: For a particular value of m,€=(n—1),(n-2),~-2,-1,0, ie. (n) values and for a particular value of J m=0,(£-1),(€-2),.. —(€-1),-£, ie., (2€+1) values. Thus degeneracy of particular en- cexgy level is, Zi 1) =n? Coulomb + Central field ‘ Non-central field m field! ; i i 12 : + t 0 1 a t —-2 4H 7 0 1 ae 9 states t forn=3 1 0 1 ' 4 ' i 1 : a \ ie 4 states ‘ REE : forn=2 \& senna a ‘ Wwww.¢ayreerendeavour. com : ' ' | ; ete I=0 ' n= ae forn=1 | Figure : Degeneracy of hydrogenic atom Example 6, Show that the most probable distance by the electron from the nucleus in the ground state of | hydrogen atomis equal to the Bor’s radius. Soln, For the ground state of hydrogen atom, n= 1, €= 0 and m= 0. Hence the ground state wave fiuiction is, \n Ly Yia0-| “a | ae (may Since, the wave function s independent of @ and ¢, therefore, q@ Three Dimensional Potentials Radial probability density= P(r’ J=|ioul? 4ar? For most probable distance, P(r) isto be the maximum E(ee2a| ae al P| 2 |e 4 are 0 | 9 5 ( =0=r=4, al \ a0) ° ‘Thus the most probable distance of the electron from the nucleus of hydrogen atom in the ground state is equal tothe Bohr radius (ap ). Example 7. The normalised ground state wave function of a one-electron atom is given by rr an( 2)" v(,8,9)=2° “(2) €7' here the notations have usual meaning, Evaluate the probability of the Soln. | LOAREER ENDEAVOUR) chuamecrmitacner Tse ‘Therefore, the, proba eg the electron inte normalised en state of one-electronatomat a argerendeavour. A Zz Example 8. Ifthe ground state wave function for the hydrogen atom is distnce atomat adistance greater th yet en ve a show that average distance of the electron from the mucleus is 1.5a,. Soln. We have the average distance of the electron from the hydrogen nucleus in the ground state is, ria, LL grange 5 fre? 21/00 2 sin Odrd dd 7a | \3 Sol Sol | uals | three Dimensional Potentials 3) Using the standard integral Jetx'ae =T, we get ee “var naan] LLL na 9 a a, (2/a,) 2qual 3 ‘Therefore, (") = Faq =1 Say Example9. Find the expectation value of potential energy of the electron in the hydrogen atom in the 1s Ate state. The Wave function for the electron is Is state is given by ofthe ® Where, @y = isthe Bobrradius, Soln, t ata Soln, nee é laeniey But, o6t | 2 sin 1 sind 00° 00” rein? 0 Og? a8 Since Vio depends on r only, the terms in = Sand 3p ate ze. jend( 23) |e : Soln, Soin, Soin. Therefore, (Ex) = Example 11. Estimate the uncertainity in the radial position of the electron in the ground state of hydrogen atom, For ground state, Uncertainity in the ra nanos soeeo e Example 12. Find o i ofa hydrogen-like at ‘The radial probabil Ry =constant x exp| , wil be maximum ‘yownw, cateerendeavour. com $x gcinda| 27-22" Jo dr \ % a ar Similarly, dP, / dr =0 gives r=9a,/Z Ingeneral, r,,, =n°a,/Z Example 13. A positron and an electron forma shoritived atom called positronium before the two annihilate to produce gamma rays. Calculate, in electron vols, the ground state energy of positronium, ‘The positron has a charge +e snd mass equal to the electron mass. tials ‘rogen ‘bitals ihilate ‘Three Dimensional Potentials Soln. Reduced mass for the positronium atom, is met, My 2m, 2 where m, is the electron mass. Since, the energy for the bydrogen like atoms is directly proportional to the reduced mass. Therefore, energy of the positronium atom is half the energy of hyécogen atom. 6 Therefore, the ground state energy for the positronium atom is ~ Ber =~6.8eV Example 14, At time ¢= 0, the wave function for the hydrogen atom is 1 7,9) = = 2Vi00 + Vato + Y2Vn + V3V 2,4 ver,0) Tio 10 + V: Ms: War) ‘where the subscripts are values of the quantum numbers m, /, m. (i) What is the expectation value for the energy of the system? (ii) What is the probability of finding the system with /= 1, m= 1? (i) The expectation 1 =A (48, +E, +2 yb Es +2 Since E, = -13.8 @) CAREER ENRE: (ii) The required prSbabiiyispebysBny (21 www. careerendeavour. com Less 4e

You might also like